NUR1211 EXAM 2

Réussis tes devoirs et examens dès maintenant avec Quizwiz!

A gardener sustained a deep laceration while working and requires sutures. The patient is asked about the date of her last tetanus shot, which is over 10 years ago. Based on this information, the patient will receive a tetanus immunization. The tetanus injection will allow for the release of what? A) Antibodies B) Antigens C) Cytokines D) Phagocytes

A) Antibodies Immunizations activate the humoral immune response, culminating in antibody production.

An infection control nurse is presenting an inservice reviewing the immune response. The nurse describes the clumping effect that occurs when an antibody acts like a cross-link between two antigens. What process is the nurse explaining? A) Agglutination B) Cellular immune response C) Humoral response D) Phagocytic immune response

A) Agglutination Agglutination refers to the clumping effect occurring when an antibody acts as a cross-link between two antigens. This takes place within the context of the humoral immune response, but is not synonymous with it.

The nurse should recognize the greatest risk for the development of blindness in which of the following patients? A) A 58-year-old Caucasian woman with macular degeneration B) A 28-year-old Caucasian man with astigmatism C) A 58-year-old African American woman with hyperopia D) A 28-year-old African American man with myopia

A) A 58-year-old Caucasian woman with macular degeneration The most common causes of blindness and visual impairment among adults 40 years of age or older are diabetic retinopathy, macular degeneration, glaucoma, and cataracts.

Health care professionals are involved in the promotion of health as much as in the treatment of disease. Health promotion has evolved as a part of health care for many reasons. Which of the following factors has most influenced the growing emphasis on health promotion? A) A changing definition of health B) An awareness that wellness exists C) An expanded definition of chronic illness D) A belief that disease is preventable

A) A changing definition of health The concept of health promotion has evolved because of a changing definition of health and an awareness that wellness exists at many levels of functioning.

Nurses who are providing patient education often use motivators for learning with patients who are struggling with behavioral changes necessary to adhere to a treatment regimen. When working with a 15-year-old boy who has diabetes, which of the following motivators is most likely to be effective? A) A learning contract B) A star chart C) A point system D) A food-reward system

A) A learning contract Using a learning contract or agreement can also be a motivator for learning. Such a contract is based on assessment of patient needs; health care data; and specific, measurable goals.

During a mumps outbreak at a local school, a patient, who is a school teacher, is exposed. She has previously been immunized for mumps. What type of immunity does she possess? A) Acquired immunity B) Natural immunity C) Phagocytic immunity D) Humoral immunity

A) Acquired immunity Acquired immunity usually develops as a result of prior exposure to an antigen, often through immunization.

A patient with HIV infection has begun experiencing severe diarrhea. What is the most appropriate nursing intervention to help alleviate the diarrhea? A) Administer antidiarrheal medications on a scheduled basis, as ordered. B) Encourage the patient to eat three balanced meals and a snack at bedtime. C) Increase the patients oral fluid intake. D) Encourage the patient to increase his or her activity level.

A) Administer antidiarrheal medications on a scheduled basis, as ordered. Administering antidiarrheal agents on a regular schedule may be more beneficial than administering them on an as-needed basis, provided the patients diarrhea is not caused by an infectious microorganism.

A hospital patient is immunocompromised because of stage 3 HIV infection and the physician has ordered a chest radiograph. How should the nurse most safely facilitate the test? A) Arrange for a portable x-ray machine to be used. B) Have the patient wear a mask to the x-ray department. C) Ensure that the radiology department has been disinfected prior to the test. D) Send the patient to the x-ray department, and have the staff in the department wear masks.

A) Arrange for a portable x-ray machine to be used. A patient who is immunocompromised is at an increased risk of contracting nosocomial infections due to suppressed immunity. The safest way the test can be facilitated is to have a portable x-ray machine in the patients room.

A patient with mastoiditis is admitted to the post-surgical unit after undergoing a radical mastoidectomy. The nurse should identify what priority of postoperative care? A) Assessing for mouth droop and decreased lateral eye gaze B) Assessing for increased middle ear pressure and perforated ear drum C) Assessing for gradual onset of conductive hearing loss and nystagmus D) Assessing for scar tissue and cerumen obstructing the auditory canal

A) Assessing for mouth droop and decreased lateral eye gaze The facial nerve runs through the middle ear and the mastoid; therefore, there is risk of injuring this nerve during a mastoidectomy. When injury occurs, the patient may display mouth droop and decreased lateral gaze on the operative side.

An HIV-infected patient presents at the clinic for a scheduled CD4+ count. The results of the test are 45 cells/mL, and the nurse recognizes the patients increased risk for Mycobacterium aviumcomplex (MAC disease). The nurse should anticipate the administration of what drug? A) Azithromycin B) Vancomycin C) Levofloxacin D) Fluconazole

A) Azithromycin HIV-infected adults and adolescents should receive chemoprophylaxis against disseminatedMycobacterium avium complex (MAC disease) if they have a CD4+ count less than 50 cells/L. Azithromycin (Zithromax) or clarithromycin (Biaxin) are the preferred prophylactic agents. Vancomycin, levofloxacin, and fluconazole are not prophylactic agents for MAC.

A patient has undergone treatment for septic shock and received high doses of numerous antibiotics during the course of treatment. When planning the patients subsequent care, the nurse should be aware of what potential effect on the patients immune function? A) Bone marrow suppression B) Uncontrolled apoptosis C) Thymus atrophy D) Lymphoma

A) Bone marrow suppression Large doses of antibiotics can precipitate bone marrow suppression, affecting immune function. Antibiotics are not noted to cause apoptosis, thymus atrophy, or lymphoma.

A parish nurse is describing the relationships between health and physical fitness to a group of older adults who all attend the same church. What potential benefits of a regular exercise program should the nurse describe? Select all that apply. A) Decreased cholesterol levels B) Delayed degenerative changes C) Improved sensory function D) Improved overall muscle strength E) Increased blood sugar levels

A) Decreased cholesterol levels B) Delayed degenerative changes D) Improved overall muscle strength Clinicians and researchers who have examined the relationship between health and physical fitness have found that a regular exercise program can promote health in the following ways: by decreasing cholesterol and low-density lipoprotein levels; delaying degenerative changes, such as osteoporosis; and improving flexibility and overall muscle strength and endurance.

A gerontologic nurse has observed that patients often fail to adhere to a therapeutic regimen. What strategy should the nurse adopt to best assist an older adult in adhering to a therapeutic regimen involving wound care? A) Demonstrate a dressing change and allow the patient to practice. B) Provide a detailed pamphlet on a dressing change. C) Verbally instruct the patient how to change a dressing and check for comprehension. D) Delegate the dressing change to a trusted family member.

A) Demonstrate a dressing change and allow the patient to practice. The nurse must consider that older adults may have deficits in the ability to draw inferences, apply information, or understand major teaching points. Demonstration and practice are essential in meeting their learning needs.

A patients ocular tumor has necessitated enucleation and the patient will be fitted with a prosthesis. The nurse should address what nursing diagnosis when planning the patients discharge education? A) Disturbed body image B) Chronic pain C) Ineffective protection D) Unilateral neglect

A) Disturbed body image The use of an ocular prosthesis is likely to have a significant impact on a patients body image. Prostheses are not associated with chronic pain or ineffective protection.

A public health nurse is planning educational interventions that are based on Beckers Health Belief Model. When identifying the variables that affect local residents health promotion behaviors, what question should the nurse seek to answer? A) Do residents believe that they have ready access to health promotion resources? B) Why have previous attempts at health promotion failed? C) How much funding is available for health promotion in the community? D) Who is available to provide health promotion education in the local area?

A) Do residents believe that they have ready access to health promotion resources? Barriers, Beckers second variable, are defined as factors leading to unavailability or difficulty in gaining access to a specific health promotion alternative.

The nurse has taken shift report on her patients and has been told that one patient has an ocular condition that has primarily affected the rods in his eyes. Considering this information, what should the nurse do while caring for the patient? A) Ensure adequate lighting in the patients room. B) Provide a dimly lit room to aid vision by limiting contrast. C) Carefully point out color differences for the patient. D) Carefully point out fine details for the patient.

A) Ensure adequate lighting in the patients room. The nurse should provide adequate lighting in the patients room, as the rods are mainly responsible for night vision or vision in low light.

A patient has had a sudden loss of vision after head trauma. How should the nurse best describe the placement of items on the dinner tray? A) Explain the location of items using clock cues. B) Explain that each of the items on the tray is clearly separated. C) Describe the location of items from the bottom of the plate to the top. D) Ask the patient to describe the location of items before confirming their location.

A) Explain the location of items using clock cues. The food trays composition is likened to the face of a clock.

The nurse in the ED is caring for a 4 year-old brought in by his parents who state that the child will not stop crying and pulling at his ear. Based on information collected by the nurse, which of the following statements applies to a diagnosis of external otitis? A) External otitis is characterized by aural tenderness. B) External otitis is usually accompanied by a high fever. C) External otitis is usually related to an upper respiratory infection. D) External otitis can be prevented by using cotton-tipped applicators to clean the ear.

A) External otitis is characterized by aural tenderness. Patients with otitis externa usually exhibit pain, discharge from the external auditory canal, and aural tenderness. Fever and accompanying upper respiratory infection occur more commonly in conjunction with otitis media (infection of the middle ear). Cotton-tipped applicators can actually cause external otitis so their use should be avoided.

A nurse recognizes that individuals of different ages have specific health promotion needs. When planning to promote health among young adults, what subject is most likely to meet this demographic groups learning needs? A) Family planning B) Management of risky behaviors C) Physical fitness D) Relationship skills training

A) Family planning Because of the nationwide emphasis on health during the reproductive years, young adults actively seek programs that address prenatal health, parenting, family planning, and womens health issues.

A patient who has been newly diagnosed with systemic lupus erythematosus (SLE) has been admitted to the medical unit. Which of the following nursing diagnoses is the most plausible inclusion in the plan of care? A) Fatigue Related to Anemia B) Risk for Ineffective Tissue Perfusion Related to Venous Thromboembolism C) Acute Confusion Related to Increased Serum Ammonia Levels D) Risk for Ineffective Tissue Perfusion Related to Increased Hematocrit

A) Fatigue Related to Anemia Patients with SLE nearly always experience fatigue, which is partly attributable to anemia.

The nurse is admitting a 55-year-old male patient diagnosed with a retinal detachment in his left eye. While assessing this patient, what characteristic symptom would the nurse expect to find? A) Flashing lights in the visual field B) Sudden eye pain C) Loss of color vision D) Colored halos around lights

A) Flashing lights in the visual field Flashing lights in the visual field is a common symptom of retinal detachment.

Since the emergence of HIV/AIDS, there have been significant changes in epidemiologic trends. Members of what group currently have the greatest risk of contracting HIV? A) Gay, bisexual, and other men who have sex with men B) Recreational drug users C) Blood transfusion recipients D) Health care providers

A) Gay, bisexual, and other men who have sex with men Gay, bisexual, and other men who have sex with men remain the population most affected by HIV and account for 2% of the population but 61% of the new infections.

Health promotion ranks high on the list of health-related concerns of the American public. Based on current knowledge, what factor should the nurse prioritize in an effort to promote health, longevity, and weight control in patients? A) Good nutrition B) Stress reduction C) Use of vitamins D)Screening for health risks

A) Good nutrition It has been suggested that good nutrition is the single most significant factor in determining health status, longevity, and weight control. A balanced diet that uses few artificial ingredients and is low in fat, caffeine, and sodium constitutes a healthy diet.

A clinic nurse is caring for a patient admitted with AIDS. The nurse has assessed that the patient is experiencing a progressive decline in cognitive, behavioral, and motor functions. The nurse recognizes that these symptoms are most likely related to the onset of what complication? A) HIV encephalopathy B) B-cell lymphoma C) Kaposis sarcoma D) Wasting syndrome

A) HIV encephalopathy HIV encephalopathy is a clinical syndrome characterized by a progressive decline in cognitive, behavioral, and motor functions.

A 6-year-old child is brought to the pediatric clinic for the assessment of redness and discharge from the eye and is diagnosed with viral conjunctivitis. What is the most important information to discuss with the parents and child? A) Handwashing can prevent the spread of the disease to others. B) The importance of compliance with antibiotic therapy C) Signs and symptoms of complications, such as meningitis and septicemia D) The likely need for surgery to prevent scarring of the conjunctiva

A) Handwashing can prevent the spread of the disease to others. The nurse must inform the parents and child that viral conjunctivitis is highly contagious and instructions should emphasize the importance of handwashing and avoiding sharing towels, face cloths, and eye drops.

A nurse is planning an educational event for a local group of citizens who live with a variety of physical and cognitive disabilities. What variable should the nurse prioritize when planning this event? A) Health-promotion needs of the group. B) Relationships between participants and caregivers . C) Wellness state of each individual D) Learning needs of caregivers

A) Health-promotion needs of the group. The nurse must be aware of the participants specific health-promotion needs when teaching specific groups of people with physical and mental disabilities.

A patients exposure to which of the following microorganisms is most likely to trigger a cellular response? A) Herpes simplex B) Staphylococcus aureus C) Pseudomonas aeruginosa D) Beta hemolytic Streptococcus

A) Herpes simplex Viral, rather than bacterial antigens, induce a cellular response.

Middle-aged adults are part of an age group that is known to be interested in health and health promotion, and the nurse is planning health promotion activities accordingly. To what suggestions do members of this age group usually respond with enthusiasm? Select all that apply. A) How lifestyle practices can improve health B) How to eat healthier C) How exercise can improve your life D) Strategies for adhering to prescribed therapy E) Exercise for the aging

A) How lifestyle practices can improve health B) How to eat healthier C) How exercise can improve your life Young and middle-aged adults represent an age group that not only expresses an interest in health and health promotion but also responds enthusiastically to suggestions that show how lifestyle practices can improve health; these lifestyle practices include nutrition and exercise. Middle-aged adults may not respond positively to teaching aimed at the aging. Adherence is not noted to be a desired focus in this age group.

A nurse is working with a patient with rheumatic disease who is being treated with salicylate therapy. What statement would indicate that the patient is experiencing adverse effects of this drug? A) I have this ringing in my ears that just wont go away. B) I feel so foggy in the mornings and it takes me so long to wake up. C) When I eat a meal thats high in fat, I get really nauseous. D) I seem to have lost my appetite, which is unusual for me.

A) I have this ringing in my ears that just wont go away. Tinnitus is associated with salicylate therapy. Salicylates do not normally cause drowsiness, intolerance of high-fat meals, or anorexia.

A patient with a history of dermatitis takes corticosteroids on a regular basis. The nurse should assess the patient for which of the following complications of therapy? A) Immunosuppression B) Agranulocytosis C) Anemia D) Thrombocytopenia

A) Immunosuppression Corticosteroids such as prednisone can cause immunosuppression. Corticosteroids do not typically cause agranulocytosis, anemia, or low platelet counts.

A nurse is planning the care of a patient with AIDS who is admitted to the unit withPneumocystis pneumonia (PCP). Which nursing diagnosis has the highest priority for this patient? A) Ineffective Airway Clearance B) Impaired Oral Mucous Membranes C) Imbalanced Nutrition: Less than Body Requirements D) Activity Intolerance

A) Ineffective Airway Clearance Although all these nursing diagnoses are appropriate for a patient with AIDS, Ineffective Airway Clearance is the priority nursing diagnosis for the patient with Pneumocystis pneumonia (PCP). Airway and breathing take top priority over the other listed concerns.

A nurse is planning the care of a patient who has a long history of chronic pain, which has only recently been diagnosed as fibromyalgia. What nursing diagnosis is most likely to apply to this womans care needs? A) Ineffective Role Performance Related to Pain B) Risk for Impaired Skin Integrity Related to Myalgia C) Risk for Infection Related to Tissue Alterations D) Unilateral Neglect Related to Neuropathic Pain

A) Ineffective Role Performance Related to Pain Typically, patients with fibromyalgia have endured their symptoms for a long period of time. The neuropathic pain accompanying FM can often impair a patients ability to perform normal roles and functions.

A nurse is teaching a patient with glaucoma how to administer eye drops to achieve maximum absorption. The nurse should teach the patient to perform what action? A) Instill the medication in the conjunctival sac. B) Maintain a supine position for 10 minutes after administration. C) Keep the eyes closed for 1 to 2 minutes after administration. D) Apply the medication evenly to the sclera

A) Instill the medication in the conjunctival sac. Eye drops should be instilled into the conjunctival sac, where absorption can best take place, rather than distributed over the sclera.

A woman has been diagnosed with breast cancer and is being treated aggressively with a chemotherapeutic regimen. As a result of this regimen, she has an inability to fight infection due to the fact that her bone marrow is unable to produce a sufficient amount of what? A) Lymphocytes B) Cytoblasts C) Antibodies D) Capillaries

A) Lymphocytes

The advanced practice nurse is attempting to examine the patients ear with an otoscope. Because of impacted cerumen, the tympanic membrane cannot be visualized. The nurse irrigates the patients ear with a solution of hydrogen peroxide and water to remove the impacted cerumen. What nursing intervention is most important to minimize nausea and vertigo during the procedure? A) Maintain the irrigation fluid at a warm temperature. B) Instill short, sharp bursts of fluid into the ear canal. C) Follow the procedure with insertion of a cerumen curette to extract missed ear wax. D) Have the patient stand during the procedure.

A) Maintain the irrigation fluid at a warm temperature. Warm water (never cold or hot) and gentle, not forceful, irrigation should be used to remove cerumen. Too forceful irrigation can cause perforation of the tympanic membrane, and ice water causes vomiting.

A nurse is addressing the incidence and prevalence of HIV infection among older adults. What principle should guide the nurses choice of educational interventions? A) Many older adults do not see themselves as being at risk for HIV infection. B) Many older adults are not aware of the difference between HIV and AIDS. C) Older adults tend to have more sex partners than younger adults. D) Older adults have the highest incidence of intravenous drug use.

A) Many older adults do not see themselves as being at risk for HIV infection. It is known that many older adults do not see themselves as being at risk for HIV infection. Knowledge of the relationship between HIV infection and AIDS is not known to affect the incidence of new cases. The statements about sex partners and IV drug use are untrue.

A patient is undergoing diagnostic testing to determine the etiology of recent joint pain. The patient asks the nurse about the difference between osteoarthritis (OA) and rheumatoid arthritis (RA). What is the best response by the nurse? A) OA is a considered a noninflammatory joint disease. RA is characterized by inflamed, swollen joints. B) OA and RA are very similar. OA affects the smaller joints such as the fingers, and RA affects the larger, weight-bearing joints like the knees. C) OA originates with an infection. RA is a result of your body's cells attacking one another. D) OA is associated with impaired immune function; RA is a consequence of physical damage.

A) OA is a considered a noninflammatory joint disease. RA is characterized by inflamed, swollen joints. OA is a degenerative arthritis with a noninflammatory etiology, characterized by the loss of cartilage on the articular surfaces of weight-bearing joints, with spur development.

A patient got a sliver of glass in his eye when a glass container at work fell and shattered. The glass had to be surgically removed and the patient is about to be discharged home. The patient asks the nurse for a topical anesthetic for the pain in his eye. What should the nurse respond? A) Overuse of these drops could soften your cornea and damage your eye. B) You could lose the peripheral vision in your eye if you used these drops too much. C) Im sorry, this medication is considered a controlled substance and patients cannot take it home. D) I know these drops will make your eye feel better, but I cant let you take them home.

A) Overuse of these drops could soften your cornea and damage your eye. Most patients are not allowed to take topical anesthetics home because of the risk of overuse. Patients with corneal abrasions and erosions experience severe pain and are often tempted to overuse topical anesthetic eye drops. Overuse of these drops results in softening of the cornea. Prolonged use of anesthetic drops can delay wound healing and can lead to permanent corneal opacification and scarring, resulting in visual loss. The nurse must explain the rationale for limiting the home use of these medications.

A hearing-impaired patient is scheduled to have an MRI. What would be important for the nurse to remember when caring for this patient? A) Patient is likely unable to hear the nurse during test. B) A person adept in sign language must be present during test. C) Lip reading will be the method of communication that is necessary. D) The nurse should interact with the patient like any other patient.

A) Patient is likely unable to hear the nurse during test. During health care and screening procedures, the practitioner (e.g., dentist, physician, nurse) must be aware that patients who are deaf or hearing-impaired are unable to read lips, see a signer, or read written materials in the dark rooms required during some diagnostic tests.

34. A nurse is assessing the skin integrity of a patient who has AIDS. When performing this inspection, the nurse should prioritize assessment of what skin surfaces? A) Perianal region and oral mucosa B) Sacral region and lower abdomen C) Scalp and skin over the scapulae D) Axillae and upper thorax

A) Perianal region and oral mucosa The nurse should inspect all the patients skin surfaces and mucous membranes, but the oral mucosa and perianal region are particularly vulnerable to skin breakdown and fungal infection.

The nurse is teaching a local community group about the importance of disease prevention. Why is the nurse justified in emphasizing disease prevention as a component of health promotion? A) Prevention is emphasized as the link between personal behavior and health. B) Most Americans die of preventable causes. C) Health maintenance organizations (HMOs) now emphasize prevention as the main criterion of health care. D) External environment affects the outcome of most disease processes.

A) Prevention is emphasized as the link between personal behavior and health. Healthy People 2020 defines the current national health-promotion and disease-prevention initiative for the nation.

The nurses plan of care for a patient with stage 3 HIV addresses the diagnosis of Risk for Impaired Skin Integrity Related to Candidiasis. What nursing intervention best addresses this risk? A) Providing thorough oral care before and after meals B) Administering prophylactic antibiotics C) Promoting nutrition and adequate fluid intake D) Applying skin emollients as needed

A) Providing thorough oral care before and after meals Thorough mouth care has the potential to prevent or limit the severity of this infection. Antibiotics are

The nurse is preparing to care for a patient who has scleroderma. The nurse refers to resources that describe CREST syndrome. Which of the following is a component of CREST syndrome? A) Raynauds phenomenon B) Thyroid dysfunction C) Esophageal varices D) Osteopenia

A) Raynauds phenomenon The R in CREST stands for Raynauds phenomenon.

A patients decreased mobility is ultimately the result of an autoimmune reaction originating in the synovial tissue, which caused the formation of pannus. This patient has been diagnosed with what health problem? A) Rheumatoid arthritis (RA) B) Systemic lupus erythematosus C) Osteoporosis D) Polymyositis

A) Rheumatoid arthritis (RA) In RA, the autoimmune reaction results in phagocytosis, producing enzymes within the joint that break down collagen, cause edema and proliferation of the synovial membrane, and ultimately form pannus.

A patient with otosclerosis has significant hearing loss. What should the nurse do to best facilitate communication with the patient? A) Sit or stand in front of the patient when speaking. B) Use exaggerated lip and mouth movements when talking. C) Stand in front of a light or window when speaking. D) Say the patients name loudly before starting to talk.

A) Sit or stand in front of the patient when speaking. Standing directly in front of a hearing-impaired patient allows him or her to lip-read and see facial expressions that offer clues to what is being said.

The nurse is providing care for a patient who has multiple sclerosis. The nurse recognizes the autoimmune etiology of this disease and the potential benefits of what treatment? A) Stem cell transplantation B) Serial immunizations C) Immunosuppression D) Genetic engineering

A) Stem cell transplantation Clinical trials using stem cells are under way in patients with a variety of disorders having an autoimmune component, including multiple sclerosis.

The nurse is providing discharge education for a patient with a new diagnosis of Mnires disease. What food should the patient be instructed to limit or avoid? A) Sweet pickles B) Frozen yogurt C) Shellfish D) Red meat

A) Sweet pickles The patient with Mnires disease should avoid foods high in salt and/or sugar; sweet pickles are high in both.

A patient who has AIDS is being treated in the hospital and admits to having periods of extreme anxiety. What would be the most appropriate nursing intervention? A) Teach the patient guided imagery. B) Give the patient more control of her antiretroviral regimen. C) Increase the patients activity level. D) Collaborate with the patients physician to obtain an order for hydromorphone.

A) Teach the patient guided imagery. Measures such as relaxation and guided imagery may be beneficial because they decrease anxiety, which contributes to weakness and fatigue.

A patient diagnosed with arthritis has been taking aspirin and now reports experiencing tinnitus and hearing loss. What should the nurse teach this patient? A) The hearing loss will likely resolve with time after the drug is discontinued. B) The patients hearing loss and tinnitus are irreversible at this point. C) The patients tinnitus is likely multifactorial, and not directly related to aspirin use. D) The patients tinnitus will abate as tolerance to aspirin develops.

A) The hearing loss will likely resolve with time after the drug is discontinued. Tinnitus and hearing loss are signs of ototoxicity, which is associated with aspirin use. In most cases, this will resolve upon discontinuing the aspirin. Many other drugs cause irreversible ototoxicity.

You are the oncoming nurse and you have just taken end-of-shift report on your patients. One of your patients newly diagnosed with diabetes was admitted with diabetic ketoacidosis. Which behavior best demonstrates this patients willingness to learn? A) The patient requests a visit from the hospitals diabetic educator. B) The patient sets aside a dessert brought in by a family member. C) The patient wants a family member to meet with the dietician to discuss meals. D) The patient readily allows the nurse to measure his blood glucose level.

A) The patient requests a visit from the hospitals diabetic educator. Emotional readiness also affects the motivation to learn. A person who has not accepted an existing illness or the threat of illness is not motivated to learn.

A patient with glaucoma has presented for a scheduled clinic visit and tells the nurse that she has begun taking an herbal remedy for her condition that was recommended by a work colleague. What instruction should the nurse provide to the patient? A) The patient should discuss this new remedy with her ophthalmologist promptly. B) The patient should monitor her IOP closely for the next several weeks. C) The patient should do further research on the herbal remedy. D) The patient should report any adverse effects to her pharmacist.

A) The patient should discuss this new remedy with her ophthalmologist promptly. Patients should discuss any new treatments with an ophthalmologist; this should precede the patients own further research or reporting adverse effects to the pharmacist.

A nurse is caring for a 78-year-old patient with a history of osteoarthritis (OA). When planning the patients care, what goal should the nurse include? A) The patient will express satisfaction with her ability to perform ADLs. B) The patient will recover from OA within 6 months. C) The patient will adhere to the prescribed plan of care. D) The patient will deny signs or symptoms of OA.

A) The patient will express satisfaction with her ability to perform ADLs. Pain management and optimal functional ability are major goals of nursing interventions for OA. Cure is not a possibility and it is unrealistic to expect a complete absence of signs and symptoms. Adherence to the plan of care is highly beneficial, but this is not the priority goal of care.

A patient is vigilant in her efforts to take good care of herself but is frustrated by her recent history of upper respiratory infections and influenza. What aspect of the patients lifestyle may have a negative effect on immune response? A) The patient works out at the gym twice daily. B) The patient does not eat red meats. C) The patient takes over-the-counter dietary supplements. D) The patient sleeps approximately 6 hours each night.

A) The patient works out at the gym twice daily. Rigorous exercise or competitive exerciseusually considered a positive lifestyle factorcan be a physiologic stressor and cause negative effects on immune response.

An advanced practice nurse has performed a Rinne test on a new patient. During the test, the patient reports that air-conducted sound is louder than bone-conducted sound. How should the nurse best interpret this assessment finding? A) The patients hearing is likely normal. B) The patient is at risk for tinnitus. C) The patient likely has otosclerosis. D) The patient likely has sensorineural hearing loss.

A) The patients hearing is likely normal. The Rinne test is useful for distinguishing between conductive and sensorineural hearing loss. A person with normal hearing reports that air-conducted sound is louder than bone-conducted sound.

The nurse is discussing the results of a patients diagnostic testing with the nurse practitioner. What Weber test result would indicate the presence of a sensorineural loss? A) The sound is heard better in the ear in which hearing is better. B) The sound is heard equally in both ears. C) The sound is heard better in the ear in which hearing is poorer. D) The sound is heard longer in the ear in which hearing is better.

A) The sound is heard better in the ear in which hearing is better. A patient with sensorineural hearing loss hears the sound better in the ear in which hearing is better. The Weber test assesses bone conduction of sound and is used for assessing unilateral hearing loss.

A public health nurse is assessing the nutritional awareness of a group of women who are participating in a prenatal health class. What outcome would most clearly demonstrate that the women possess nutritional awareness? A) The women demonstrate an understanding of the importance of a healthy diet. B) The women are able to describe the importance of vitamin supplements during pregnancy. C) The women can list the minerals nutrients that should be consumed daily. D) The women can interpret the nutrition facts listed on food packaging.

A) The women demonstrate an understanding of the importance of a healthy diet. Nutritional awareness involves an understanding of the importance of a healthy diet that supplies all of the essential nutrients.

A patient is postoperative day 6 following tympanoplasty and mastoidectomy. The patient has phoned the surgical unit and states that she is experiencing occasional sharp, shooting pains in her affected ear. How should the nurse best interpret this patients complaint? A) These pains are an expected finding during the first few weeks of recovery. B) The patients complaints are suggestive of a postoperative infection. C) The patient may have experienced a spontaneous rupture of the tympanic membrane. D) The patients surgery may have been unsuccessful.

A) These pains are an expected finding during the first few weeks of recovery. For 2 to 3 weeks after surgery, the patient may experience sharp, shooting pains intermittently as the eustachian tube opens and allows air to enter the middle ear.

A patient comes to the ophthalmology clinic for an eye examination. The patient tells the nurse that he often sees floaters in his vision. How should the nurse best interpret this subjective assessment finding? A) This is a normal aging process of the eye. B) Glasses will minimize this phenomenon. C) The patient may be exhibiting signs of glaucoma. D) This may be a result of weakened ciliary muscles.

A) This is a normal aging process of the eye. As the body ages, the perfect gel-like characteristics of the vitreous humor are gradually lost, and various cells and fibers cast shadows that the patient perceives as floaters.

A patient with HIV has a nursing diagnosis of Risk for Impaired Skin Integrity. What nursing intervention best addresses this risk? A) Utilize a pressure-reducing mattress. B) Limit the patients physical activity. C) Apply antibiotic ointment to dependent skin surfaces. D) Avoid contact with synthetic fabrics.

A) Utilize a pressure-reducing mattress. Devices such as alternating-pressure mattresses and low-air-loss beds are used to prevent skin breakdown.

A patient is scheduled for enucleation and the nurse is providing anticipatory guidance about postoperative care. What aspects of care should the nurse describe to the patient? Select all that apply. A) Application of topical antibiotic ointment B) Maintenance of a supine position for the first 48 hours postoperative C) Fluid restriction to prevent orbital edema D) Administration of loop diuretics to prevent orbital edema E) Use of an ocular pressure dressing

A,E A) Application of topical antibiotic ointment E) Use of an ocular pressure dressing Patients who undergo eye removal need to know that they will usually have a large ocular pressure dressing, which is typically removed after a week, and that an ophthalmic topical antibiotic ointment is applied in the socket three times daily.

You are the clinic nurse providing patient education to a teenage girl who was diagnosed 6 months ago with type 1 diabetes. Her hemoglobin A1C results suggest she has not been adhering to her prescribed treatment regimen. As the nurse, what variables do you need to assess to help this patient better adhere to her treatment regimen? Select all that apply. A) Variables that affect the patients ability to obtain resources B) Variables that affect the patients ability to teach her friends about diabetes C) Variables that affect the patients ability to cure her disease D) Variables that affect the patients ability to maintain a healthy social environment E) Variables that affect the patients ability to adopt specific behaviors

A) Variables that affect the patients ability to obtain resources D) Variables that affect the patients ability to maintain a healthy social environment E) Variables that affect the patients ability to adopt specific behaviors Nurses success with health education is determined by ongoing assessment of the variables that affect a patients ability to adopt specific behaviors, to obtain resources, and to maintain a healthy social environment.

A patient with an exacerbation of systemic lupus erythematosus (SLE) has been hospitalized on the medical unit. The nurse observes that the patient expresses anger and irritation when her call bell isn't answered immediately. What would be the most appropriate response? A) You seem like you're feeling angry. Is that something that we could talk about? B) Try to remember that stress can make your symptoms worse. C) Would you like to talk about the problem with the nursing supervisor? D) I can see you're angry. Ill come back when you've calmed down.

A) You seem like you're feeling angry. Is that something that we could talk about? The changes and the unpredictable course of SLE necessitate expert assessment skills and nursing care, as well as sensitivity to the psychological reactions of the patient. Offering to listen to the patient express anger can help the nurse and the patient understand its cause and begin to deal with it.

The nurse is providing care for a patient who has benefited from a cochlear implant. The nurse should understand that this patients health history likely includes which of the following? Select all that apply. A) The patient was diagnosed with sensorineural hearing loss. B) The patients hearing did not improve appreciably with the use of hearing aids. C) The patient has deficits in peripheral nervous function. D) The patients hearing deficit is likely accompanied by a cognitive deficit. E) The patient is unable to lip-read.

A,B A) The patient was diagnosed with sensorineural hearing loss. B) The patients hearing did not improve appreciably with the use of hearing aids. A cochlear implant is an auditory prosthesis used for people with profound sensorineural hearing loss bilaterally who do not benefit from conventional hearing aids.

A nurse is performing the admission assessment of a patient who has AIDS. What components should the nurse include in this comprehensive assessment? Select all that apply. A) Current medication regimen B) Identification of patients support system C) Immune system function D) Genetic risk factors for HIV E) History of sexual practices

A,B,C,E A) Current medication regimen B) Identification of patients support system C) Immune system function E) History of sexual practices Nursing assessment includes numerous focuses, including identification of medication use, support system, immune function and sexual history. HIV does not have a genetic component.

A nurse is completing a nutritional status of a patient who has been admitted with AIDS-related complications. What components should the nurse include in this assessment? Select all that apply. A) Serum albumin level B) Weight history C) White blood cell count D) Body mass index E) Blood urea nitrogen (BUN) level

A,B,D,E A) Serum albumin level B) Weight history D) Body mass index E) Blood urea nitrogen (BUN) level Nutritional status is assessed by obtaining a dietary history and identifying factors that may interfere with oral intake, such as anorexia, nausea, vomiting, oral pain, or difficulty swallowing. In addition, the patients ability to purchase and prepare food is assessed. Weight history (i.e., changes over time); anthropometric measurements; and blood urea nitrogen (BUN), serum protein, albumin, and transferrin levels provide objective measurements of nutritional status. White cell count is not a typical component of a nutritional assessment.

A patient has been admitted to a medical unit with a diagnosis of polymyalgia rheumatica (PMR). The nurse should be aware of what aspects of PMR? Select all that apply. A) PMR has an association with the genetic marker HLA-DR4. B) Immunoglobulin deposits occur in PMR. C) PMR is considered to be a wear-and-tear disease. D) Foods high in purines exacerbate the biochemical processes that occur in PMR. E) PMR occurs predominately in Caucasians.

A,B,E A) PMR has an association with the genetic marker HLA-DR4. B) Immunoglobulin deposits occur in PMR. E) PMR occurs predominately in Caucasians. The underlying mechanism involved with polymyalgia rheumatica is unknown. This disease occurs predominately in Caucasians and often in first-degree relatives. An association with the genetic marker HLA-DR4 suggests a familial predisposition. Immunoglobulin deposits in the walls of inflamed temporal arteries also suggest an autoimmune process.

A public health nurse is teaching a health promotion workshop that focuses on vision and eye health. What should this nurse cite as the most common causes of blindness and visual impairment among adults over the age of 40? Select all that apply. A) Diabetic retinopathy B) Trauma C) Macular degeneration D) Cytomegalovirus E) Glaucoma

A,C,E A) Diabetic retinopathy C) Macular degeneration E) Glaucoma The most common causes of blindness and visual impairment among adults 40 years of age or older are diabetic retinopathy, macular degeneration, glaucoma, and cataracts.

A nurse has administered a childs scheduled vaccination for rubella. This vaccination will cause the child to develop which of the following? A) Natural immunity B) Active acquired immunity C) Cellular immunity D) Mild hypersensitivity

B) Active acquired immunity Active acquired immunity usually develops as a result of vaccination or contracting a disease.

A nurse is working with a patient who was diagnosed with HIV several months earlier. The nurse should recognize that a patient with HIV is considered to have AIDS at the point when the CD4+ T-lymphocyte cell count drops below what threshold? A) 75 cells/mm3 of blood B) 200 cells/mm3 of blood C) 325 cells/mm3 of blood D) 450 cells/mm3 of blood

B) 200 cells/mm3 of blood When CD4+ T-cell levels drop below 200 cells/mm3 of blood, the person is said to have AIDS.

A team of public health nurses are doing strategic planning and are discussing health promotion activities for the next year. Which of the following initiatives best exemplifies the principles of health promotion? A) A blood pressure clinic at a local factory B) A family planning clinic at a community center C) An immunization clinic at the largest local mall D) A workplace safety seminar

B) A family planning clinic at a community center Health promotion may be defined as those activities that assist people in developing resources that maintain or enhance well-being and improve their quality of life.

A public health nurse is preparing to hold a series of health-promotion classes for middle-aged adults that will address a variety of topics. Which site would best meet the learning needs of this population? A) A well-respected physicians office B) A large, local workplace C) The local hospital D) An ambulatory clinic

B) A large, local workplace The workplace has become a center for health-promotion activity. Health-promotion programs can generally be offered almost anywhere in the community, but the workplace is often more convenient for the adult, working population. This makes this option preferable to a hospital, doctors office, or ambulatory clinic.

A patient who presents for an eye examination is diagnosed as having a visual acuity of 20/40. The patient asks the nurse what these numbers specifically mean. What is a correct response by the nurse? A) A person whose vision is 20/40 can see an object from 40 feet away that a person with 20/20 vision can see from 20 feet away. B) A person whose vision is 20/40 can see an object from 20 feet away that a person with 20/20 vision can see from 40 feet away. C) A person whose vision is 20/40 can see an object from 40 inches away that a person with 20/20 vision can see from 20 inches away. D) A person whose vision is 20/40 can see an object from 20 inches away that a person with 20/20 vision can see from 40 inches away.

B) A person whose vision is 20/40 can see an object from 20 feet away that a person with 20/20 vision can see from 40 feet away. The Snellen chart is a tool used to measure visual acuity. It is composed of a series of progressively smaller rows of letters and is used to test distance vision. The fraction 20/20 is considered the standard of normal vision. Most people can see the letters on the line designated as 20/20 from a distance of 20 feet. A person whose vision is 20/40 can see an object from 20 feet away that a person with 20/20 vision can see from 40 feet away.

A patient is beginning an antiretroviral drug regimen shortly after being diagnosed with HIV. What nursing action is most likely to increase the likelihood of successful therapy? A) Promoting appropriate use of complementary therapies B) Addressing possible barriers to adherence C) Educating the patient about the pathophysiology of HIV D) Teaching the patient about the need for follow-up blood work

B) Addressing possible barriers to adherence ART is highly dependent on adherence to treatment, and the nurse should proactively address this.

A nurse is explaining the process by which the body removes cells from circulation after they have performed their physiologic function. The nurse is describing what process? A) The cellular immune response B) Apoptosis C) Phagocytosis D) Opsonization

B) Apoptosis Apoptosis, or programmed cell death, is the bodys way of destroying worn out cells such as blood or skin cells or cells that need to be renewed.

The nurse is providing discharge education to an adult patient who will begin a regimen of ocular medications for the treatment of glaucoma. How can the nurse best determine if the patient is able to self-administer these medications safely and effectively? A) Assess the patient for any previous inability to self-manage medications. B) Ask the patient to demonstrate the instillation of her medications. C) Determine whether the patient can accurately describe the appropriate method of administering her medications. D) Assess the patients functional status.

B) Ask the patient to demonstrate the instillation of her medications. The patient or the caregiver at home should be asked to demonstrate actual eye drop administration. This method of assessment is more accurate than asking the patient to describe the process or determining earlier inabilities to self-administer medications.

During discharge teaching the nurse realizes that the patient is not able to read medication bottles accurately and has not been taking her medications consistently at home. How should the nurse intervene most appropriately in this situation? A) Ask the social worker to investigate alternative housing arrangements. B) Ask the social worker to investigate community support agencies. C) Encourage the patient to explore surgical corrections for the vision problem. D) Arrange for referral to a rehabilitation facility for vision training.

B) Ask the social worker to investigate community support agencies. Managing low vision involves magnification and image enhancement through the use of low-vision aids and strategies and referrals to social services and community agencies serving those with visual impairment.

A patient has lost most of her vision as a result of macular degeneration. When attempting to meet this patients psychosocial needs, what nursing action is most appropriate? A) Encourage the patient to focus on her use of her other senses. B) Assess and promote the patients coping skills during interactions with the patient. C) Emphasize that her lifestyle will be unchanged once she adapts to her vision loss. D) Promote the patients hope for recovery.

B) Assess and promote the patients coping skills during interactions with the patient. The nurse should empathically promote the patients coping with her loss. Focusing on the remaining senses could easily be interpreted as downplaying the patients loss, and recovery is not normally a realistic possibility. Even with successful adaptation, the patients lifestyle will be profoundly affected.

The nurse and a colleague are performing the Epley maneuver with a patient who has a diagnosis of benign paroxysmal positional vertigo. The nurses should begin this maneuver by performing what action? A) Placing the patient in a prone position B) Assisting the patient into a sitting position C) Instilling 15 mL of warm normal saline into one of the patients ears D) Assessing the patients baseline hearing by performing the whisper test

B) Assisting the patient into a sitting position The Epley maneuver is performed by placing the patient in a sitting position, turning the head to a 45- degree angle on the affected side, and then quickly moving the patient to the supine position.

The nurse is providing health education to a patient newly diagnosed with glaucoma. The nurse teaches the patient that this disease has a familial tendency. The nurse should encourage the patients immediate family members to undergo clinical examinations how often? A) At least monthly B) At least once every 2 years C) At least once every 5 years D) At least once every 10 years

B) At least once every 2 years Glaucoma has a family tendency and family members should be encouraged to undergo examinations at least once every 2 years to detect glaucoma early.

A patient has come into contact with HIV. As a result, HIV glycoproteins have fused with the patients CD4+ T-cell membranes. This process characterizes what phase in the HIV life cycle? A) Integration B) Attachment C) Cleavage D) Budding

B) Attachment During the process of attachment, glycoproteins of HIV bind with the hosts uninfected CD4+ receptor and chemokine coreceptors, which results in fusion of HIV with the CD4+ T-cell membrane.

The nurse is admitting a patient to the unit who is scheduled to have an ossiculoplasty. What postoperative assessment will best determine whether the procedure has been successful? A) Otoscopy B) Audiometry C) Balance testing D) Culture and sensitivity testing of ear discharge

B) Audiometry Ossiculoplasty is the surgical reconstruction of the middle ear bones to restore hearing. Consequently, results are assessed by testing hearing, not by visualizing the ear, testing balance, or culturing ear discharge.

A nurse is performing the initial assessment of a patient who has a recent diagnosis of systemic lupus erythematosus (SLE). What skin manifestation would the nurse expect to observe on inspection? A) Petechiae B) Butterfly rash C) Jaundice D) Skin sloughing

B) Butterfly rash An acute cutaneous lesion consisting of a butterfly-shaped rash across the bridge of the nose and cheeks occurs in SLE. Petechiae are pinpoint skin hemorrhages, which are not a clinical manifestation of SLE.

The nurse on the medicalsurgical unit is reviewing discharge instructions with a patient who has a history of glaucoma. The nurse should anticipate the use of what medications? A) Potassium-sparing diuretics B) Cholinergics C) Antibiotics D) Loop diuretics

B) Cholinergics Cholinergics are used in the treatment of glaucoma. The action of this medication is to increase aqueous fluid outflow by contracting the ciliary muscle and causing miosis and opening the trabecular meshwork.

An elderly female patient has come to the clinic for a scheduled follow-up appointment. The nurse learns from the patients daughter that the patient is not following the instructions she received upon discharge from the hospital last month. What is the most likely factor causing the patient not to adhere to her therapeutic regimen? A) Ethnic background of health care provider B) Costs of the prescribed regimen C) Presence of a learning disability D) Personality of the physician

B) Costs of the prescribed regimen Variables that appear to influence the degree of adherence to a prescribed therapeutic regimen include gender, race, education, illness, complexity of the regimen, and the cost of treatments.

A patient is undergoing testing to determine the overall function of her immune system. What test can be performed to evaluate the functioning of the patients cellular immune system? A) Immunoglobulin testing B) Delayed hypersensitivity skin test C) Specific antibody response D) Total serum globulin assessment

B) Delayed hypersensitivity skin test Cellular (cell-mediated) immunity tests include the delayed hypersensitivity skin test, since this immune response is specifically dependent on the cellular immune response.

The nurses assessment of a patient with significant visual losses reveals that the patient cannot count fingers. How should the nurse proceed with assessment of the patients visual acuity? A) Assess the patients vision using a Snellen chart. B) Determine whether the patient is able to see the nurses hand motion. C) Perform a detailed examination of the patients external eye structures. D) Palpate the patients periocular regions.

B) Determine whether the patient is able to see the nurses hand motion. If the patient cannot count fingers, the examiner raises one hand up and down or moves it side to side and asks in which direction the hand is moving.

A nurse is performing an admission assessment on a patient with stage 3 HIV. After assessing the patients gastrointestinal system and analyzing the data, what is most likely to be the priority nursing diagnosis? A) Acute Abdominal Pain B) Diarrhea C) Bowel Incontinence D) Constipation

B) Diarrhea Diarrhea is a problem in 50% to 60% of all AIDS patients.

A patient is scheduled to have an electronystagmography as part of a diagnostic workup for Mnires disease. What question is it most important for the nurse to ask the patient in preparation for this test? A) Have you ever experienced claustrophobia or feelings of anxiety while in enclosed spaces? B) Do you currently take any tranquilizers or stimulants on a regular basis? C) Do you have a history of falls or problems with loss of balance? D) Do you have a history of either high or low blood pressure?

B) Do you currently take any tranquilizers or stimulants on a regular basis? Electronystagmography measures changes in electrical potentials created by eye movements during induced nystagmus. Medications such as tranquilizers, stimulants, or antivertigo agents are withheld for 5 days before the test. Claustrophobia is not a significant concern associated with this test; rather, it is most often a concern for patients undergoing magnetic resonance imaging (MRI). Balance is impaired by Mnires disease; therefore, a patient history of balance problems is important, but is not relevant to test preparation.

A public health nurse is preparing an educational campaign to address a recent local increase in the incidence of HIV infection. The nurse should prioritize which of the following interventions? A) Lifestyle actions that improve immune function B) Educational programs that focus on control and prevention C) Appropriate use of standard precautions D) Screening programs for youth and young adults

B) Educational programs that focus on control and prevention Until an effective vaccine is developed, preventing HIV by eliminating and reducing risk behaviors is essential. Educational interventions are the primary means by which behaviors can be influenced.

A nurse is working with a male patient who has recently received a diagnosis of human immunodeficiency virus (HIV). When performing patient education during discharge planning, what goal should the nurse emphasize most strongly? A) Encourage the patient to exercise within his limitations. B) Encourage the patient to adhere to his therapeutic regimen. C) Appraise the patients level of nutritional awareness. D) Encourage a disease-free state.

B) Encourage the patient to adhere to his therapeutic regimen. One of the goals of patient education is to encourage people to adhere to their therapeutic regimen. This is a very important goal because if patients do not adhere to their therapeutic regimen, they will not attain their optimal level of wellness.

A patient with polymyositisis experiencing challenges with activities of daily living as a result of proximal muscle weakness. What is the most appropriate nursing action? A) Initiate a program of passive range of motion exercises B) Facilitate referrals to occupational and physical therapy C) Administer skeletal muscle relaxants as ordered D) Encourage a progressive program of weight-bearing exercise

B) Facilitate referrals to occupational and physical therapy Patients with polymyositis may have symptoms similar to those of other inflammatory diseases.

A patient has just returned to the surgical floor after undergoing a retinal detachment repair. The postoperative orders specify that the patient should be kept in a prone position until otherwise ordered. What should the nurse do? A) Call the physician and ask for the order to be confirmed. B) Follow the order because this position will help keep the retinal repair intact. C) Instruct the patient to maintain this position to prevent bleeding. D) Reposition the patient after the first dressing change.

B) Follow the order because this position will help keep the retinal repair intact. For pneumatic retinopexy, postoperative positioning of the patient is critical because the injected bubble must float into a position overlying the area of detachment, providing consistent pressure to reattach the sensory retina.

A patient has just been told by his physician that he has scleroderma. The physician tells the patient that he is going to order some tests to assess for systemic involvement. The nurse knows that priority systems to be assessed include what? A) Hepatic B) Gastrointestinal C) Genitourinary D) Neurologic

B) Gastrointestinal Assessment of systemic involvement with scleroderma requires a systems review with special attention to gastrointestinal, pulmonary, renal, and cardiac systems.

A 21-year-old male has just been diagnosed with a spondyloarthropathy. What will be a priority nursing intervention for this patient? A) Referral for assistive devices B) Teaching about symptom management C) Referral to classes to stop smoking D) Setting up an exercise program

B) Teaching about symptom management Major nursing interventions in the spondyloarthropathies are related to symptom management and maintenance of optimal functioning.

A patient presents at the ED after receiving a chemical burn to the eye. What would be the nurses initial intervention for this patient? A) Generously flush the affected eye with a dilute antibiotic solution. B) Generously flush the affected eye with normal saline or water. C) Apply a patch to the affected eye. D) Apply direct pressure to the affected eye.

B) Generously flush the affected eye with normal saline or water. Chemical burns of the eye should be immediately irrigated with water or normal saline to flush the chemical from the eye.

A patient with rheumatic disease is complaining of stomatitis. The nurse caring for the patient should further assess the patient for the adverse effects of what medications? A) Corticosteroids B) Gold-containing compounds C) Antimalarials D) Salicylate therapy

B) Gold-containing compounds Stomatitis is an adverse effect that is associated with gold therapy.

As the nurse working in a gerontology clinic, you know that some elderly people do not adhere to therapeutic regimens because of chronic illnesses that require long-term treatment by several health care providers. What is the most important consideration when dealing with this segment of the population? A) Health care professionals must know all the dietary supplements the patient is taking. B) Health care professionals must work together to provide coordinated care. C) Health care professionals may negate the efforts of another health care provider. D) Health care professionals must have a peer witness their interactions with the patient.

B) Health care professionals must work together to provide coordinated care. Above all, health care professionals must work together to provide continuous, coordinated care; otherwise, the efforts of one health care professional may be negated by those of another.

A group of high school students is attending a concert, which will be at a volume of 80 to 90 dB. What is a health consequence of this sound level? A) Hearing will not be affected by a decibel level in this range. B) Hearing loss may occur with a decibel level in this range. C) Sounds in this decibel level are not perceived to be harsh to the ear. D) Ear plugs will have no effect on these decibel levels.

B) Hearing loss may occur with a decibel level in this range. Sound louder than 80 dB is perceived by the human ear to be harsh and can be damaging to the inner ear.

A patient who has AIDS has been admitted for the treatment of Kaposis sarcoma. What nursing diagnosis should the nurse associate with this complication of AIDS? A) Risk for Disuse Syndrome Related to Kaposis Sarcoma B) Impaired Skin Integrity Related to Kaposis Sarcoma C) Diarrhea Related to Kaposis Sarcoma D) Impaired Swallowing Related to Kaposis Sarcoma

B) Impaired Skin Integrity Related to Kaposis Sarcoma Kaposis sarcoma (KS) is a disease that involves the endothelial layer of blood and lymphatic vessels.

A patient with HIV will be receiving care in the home setting. What aspect of self-care should the nurse emphasize during discharge education? A) Appropriate use of prophylactic antibiotics B) Importance of personal hygiene C) Signs and symptoms of wasting syndrome D) Strategies for adjusting antiretroviral dosages

B) Importance of personal hygiene Infection control is of high importance in patients living with HIV, thus personal hygiene is paramount.

A patient has been diagnosed with glaucoma and the nurse is preparing health education regarding the patients medication regimen. The patient states that she is eager to beat this disease and looks forward to the time that she will no longer require medication. How should the nurse best respond? A) You have a great attitude. This will likely shorten the amount of time that you need medications. B) In fact, glaucoma usually requires lifelong treatment with medications. C) Most people are treated until their intraocular pressure goes below 50 mm Hg. D) You can likely expect a minimum of 6 months of treatment.

B) In fact, glaucoma usually requires lifelong treatment with medications. Glaucoma requires lifelong pharmacologic treatment. Normal intraocular pressure is between 10 and 21 mm Hg.

The nurse is caring for a patient who has undergone a mastoidectomy. In an effort to prevent postoperative infection, what intervention should the nurse implement? A) Teach the patient about the risks of ototoxic medications. B) Instruct the patient to protect the ear from water for several weeks. C) Teach the patient to remove cerumen safely at least once per week. D) Instruct the patient to protect the ear from temperature extremes until healing is complete.

B) Instruct the patient to protect the ear from water for several weeks. To prevent infection, the patient is instructed to prevent water from entering the external auditory canal for 6 weeks.

The nurse is planning the care of a patient who is adapting to the use of a hearing aid for the first time. What is the most significant challenge experienced by a patient with hearing loss who is adapting to using a hearing aid for the first time? A) Regulating the tone and volume B) Learning to cope with amplification of background noise C) Constant irritation of the external auditory canal D) Challenges in keeping the hearing aid clean while minimizing exposure to moisture

B) Learning to cope with amplification of background noise Each of the answers represents a common problem experienced by patients using a hearing aid for the first time. However, amplification of background noise is a difficult problem to manage and is the major reason why patients stop using their hearing aid.

A nurse is providing care for a patient who has just been diagnosed as being in the early stage of rheumatoid arthritis. The nurse should anticipate the administration of which of the following? A) Hydromorphone (Dilaudid) B) Methotrexate (Rheumatrex) C) Allopurinol (Zyloprim) D) Prednisone

B) Methotrexate (Rheumatrex) In the past, a step-wise approach starting with NSAIDs was standard of care. However, evidence clearly documenting the benefits of early DMARD (methotrexate [Rheumatrex], antimalarials, leflunomide [Arava], or sulfasalazine [Azulfidine]) treatment has changed national guidelines for management.

A 16-year-old has been brought to the emergency department by his parents after falling through the glass of a patio door, suffering a laceration. The nurse caring for this patient knows that the site of the injury will have an invasion of what? A) Interferons B) Phagocytic cells C) Apoptosis D) Cytokines

B) Phagocytic cells Monocytes migrate to injury sites and function as phagocytic cells, engulfing, ingesting, and destroying greater numbers and quantities of foreign bodies or toxins than granulocytes. This occurs in response to the foreign bodies that have invaded the laceration from the dirt on the broken glass.

A patient is admitted with cellulitis and experiences a consequent increase in white blood cell count. The nurse is aware that during the immune response, pathogens are engulfed by white blood cells that ingest foreign particles. What is this process known as? A) Apoptosis B) Phagocytosis C) Antibody response D) Cellular immune response

B) Phagocytosis During the first mechanism of defense, white blood cells, which have the ability to ingest foreign particles, move to the point of attack, where they engulf and destroy the invading agents. This is known as phagocytosis.

A patient was recently exposed to infectious microorganisms and many T lymphocytes are now differentiating into killer T cells. This process characterizes what stage of the immune response? A) Effector B) Proliferation C) Response D) Recognition

B) Proliferation In the proliferation stage, T lymphocytes differentiate into cytotoxic (or killer) T cells, whereas B lymphocytes produce and release antibodies.

A man was scratched by an old tool and developed a virulent staphylococcus infection. In the course of the mans immune response, circulating lymphocytes containing the antigenic message returned to the nearest lymph node. During what stage of the immune response did this occur? A) Recognition stage B) Proliferation stage C) Response stage D) Effector stage

B) Proliferation stage In the proliferation stage, the circulating lymphocyte containing the antigenic message returns to the nearest lymph node. Once in the node, the sensitized lymphocyte stimulates some of the resident dormant T and B lymphocytes to enlarge, divide, and proliferate.

A hospital nurse has experienced percutaneous exposure to an HIV-positive patients blood as a result of a needlestick injury. The nurse has informed the supervisor and identified the patient. What action should the nurse take next? A) Flush the wound site with chlorhexidine. B) Report to the emergency department or employee health department. C) Apply a hydrocolloid dressing to the wound site. D) Follow up with the nurses primary care provider.

B) Report to the emergency department or employee health department. After initiating the emergency reporting system, the nurse should report as quickly as possible to the employee health services, the emergency department, or other designated treatment facility.

A patient with rheumatoid arthritis comes to the clinic complaining of pain in the joint of his right great toe and is eventually diagnosed with gout. When planning teaching for this patient, what management technique should the nurse emphasize? A) Take OTC calcium supplements consistently. B) Restrict consumption of foods high in purines. C) Ensure fluid intake of at least 4 liters per day. D) Restrict weight-bearing on right foot.

B) Restrict consumption of foods high in purines. Although severe dietary restriction is not necessary, the nurse should encourage the patient to restrict consumption of foods high in purines, especially organ meats.

While reviewing the health history of an older adult experiencing hearing loss the nurse notes the patient has had no trauma or loss of balance. What aspect of this patients health history is most likely to be linked to the patients hearing deficit? A) Recent completion of radiation therapy for treatment of thyroid cancer B) Routine use of quinine for management of leg cramps C) Allergy to hair coloring and hair spray D) Previous perforation of the eardrum

B) Routine use of quinine for management of leg cramps Long-term, regular use of quinine for management of leg cramps is associated with loss of hearing acuity.

A patient with HIV is admitted to the hospital because of chronic severe diarrhea. The nurse caring for this patient should expect the physician to order what drug for the management of the patients diarrhea? A) Zithromax B) Sandostatin C) Levaquin D) Biaxin

B) Sandostatin Therapy with octreotide acetate (Sandostatin), a synthetic analogue of somatostatin, has been shown to be effective in managing chronic severe diarrhea.

A nurse has planned a teachinglearning interaction that is aimed at middle school-aged students. To foster successful health education, the nurses planning should prioritize which of the following components? A) Pretesting B) Social and cultural patterns C) Patient awareness D) Measurable interventions

B) Social and cultural patterns A patients social and cultural patterns must be appropriately incorporated into the teachinglearning interaction.

The nurse should recognize a patients risk for impaired immune function if the patient has undergone surgical removal of which of the following? A) Thyroid gland B) Spleen C) Kidney D) Pancreas

B) Spleen A history of surgical removal of the spleen, lymph nodes, or thymus may place the patient at risk for impaired immune function.

A patient has undergone diagnostic testing and has been diagnosed with otosclerosis? What ear structure is primarily affected by this diagnosis? A) Malleus B) Stapes C) Incus D)Tympanic membrane

B) Stapes Otosclerosis involves the stapes and is thought to result from the formation of new, abnormal bone, especially around the oval window, with resulting fixation of the stapes.

A nurse has been studying research that examines the association between stress levels and negative health outcomes. Which relationship should underlie the educational interventions that the nurse chooses to teach? A) Stress impairs sleep patterns. B) Stress decreases immune function. C) Stress increases weight. D) Stress decreases concentration.

B) Stress decreases immune function. Studies have shown the negative effects of stress on health and a cause-and-effect relationship between stress and infectious diseases, traumatic injuries (e.g., motor vehicle crashes), and some chronic illnesses.

A nurse is providing care for a patient who has a rheumatic disorder. The nurses comprehensive assessment includes the patients mood, behavior, LOC, and neurologic status. What is this patients most likely diagnosis? A) Osteoarthritis (OA) B) Systemic lupus erythematosus (SLE) C) Rheumatoid arthritis (RA) D) Gout

B) Systemic lupus erythematosus (SLE) SLE has a high degree of neurologic involvement, and can result in central nervous system changes.

A nurse is assessing a 28-year-old man with HIV who has been admitted with pneumonia. In assessing the patient, which of the following observations takes immediate priority? A) Oral temperature of 100F B) Tachypnea and restlessness C) Frequent loose stools D) Weight loss of 1 pound since yesterday

B) Tachypnea and restlessness In prioritizing care, the pneumonia would be assessed first by the nurse. Tachypnea and restlessness are symptoms of altered respiratory status and need immediate priority.

A hospitalized patient with impaired vision must get a picture in his or her mind of the hospital room and its contents in order to mobilize independently and safely. What must the nurse monitor in the patients room? A) That a commode is always available at the bedside B) That all furniture remains in the same position C) That visitors do not leave items on the bedside table D) That the patients slippers stay under the bed

B) That all furniture remains in the same position All articles and furniture must remain in the same positions throughout the patients hospitalization. This will reduce the patients risks for falls.

A nurse is explaining how the humoral and cellular immune responses should be seen as interacting parts of the broader immune system rather than as independent and unrelated processes. What aspect of immune function best demonstrates this? A) The movement of B cells in and out of lymph nodes B) The interactions that occur between T cells and B cells C) The differentiation between different types of T cells D) The universal role of the complement system

B) The interactions that occur between T cells and B cells T cells interact closely with B cells, indicating that humoral and cellular immune responses are not separate, unrelated processes, but rather branches of the immune response that interact.

A patient is in the primary infection stage of HIV. What is true of this patients current health status? A) The patients HIV antibodies are successfully, but temporarily, killing the virus. B) The patient is infected with HIV but lacks HIV-specific antibodies. C) The patients risk for opportunistic infections is at its peak. D) The patient may or may not develop long-standing HIV infection.

B) The patient is infected with HIV but lacks HIV-specific antibodies. The period from infection with HIV to the development of HIV-specific antibodies is known as primary infection.

A nurse is planning preoperative teaching for a patient with hearing loss due to otosclerosis. The patient is scheduled for a stapedectomy with insertion of a prosthesis. What information is most crucial to include in the patients preoperative teaching? A) The procedure is an effective, time-tested treatment for sensory hearing loss. B) The patient is likely to experience resolution of conductive hearing loss after the procedure. C) Several months of post-procedure rehabilitation will be needed to maximize benefits. D) The procedure is experimental, but early indications suggest great therapeutic benefits.

B) The patient is likely to experience resolution of conductive hearing loss after the procedure. Stapedectomy is a very successful time-tested procedure, resulting in the restoration of conductive hearing loss.

The nurse is assessing a clients risk for impaired immune function. What assessment finding should the nurse identify as a risk factor for decreased immunity? A) The patient takes a beta blocker for the treatment of hypertension. B) The patient is under significant psychosocial stress. C) The patient had a pulmonary embolism 18 months ago. D) The patient has a family history of breast cancer.

B) The patient is under significant psychosocial stress. Stress is a psychoneuroimmunologic factor that is known to depress the immune response. Use of beta blockers, a family history of cancer, and a prior PE are significant assessment findings, but none represents an immediate threat to immune function.

A nurse is assessing a patient for risk factors known to contribute to osteoarthritis. What assessment finding would the nurse interpret as a risk factor? A) The patient has a 30 pack-year smoking history. B) The patients body mass index is 34 (obese). C) The patient has primary hypertension. D) The patient is 58 years old.

B) The patients body mass index is 34 (obese). Risk factors for osteoarthritis include obesity and previous joint damage. Risk factors of OA do not include smoking or hypertension.

A community health nurse has been asked to participate in a health fair that is being sponsored by the local senior center. The nurse should select educational focuses in the knowledge that older adults benefit most from what kind of activities? A) Those that help them eat well B) Those that help them maintain independence C) Those that preserve their social interactions D) Those that promote financial stability

B) Those that help them maintain independence Although their chronic illnesses and disabilities cannot be eliminated, the elderly can benefit most from activities that help them maintain independence and achieve an optimal level of health.

A nurse is planning patient education for a patient being discharged home with a diagnosis of rheumatoid arthritis. The patient has been prescribed antimalarials for treatment, so the nurse knows to teach the patient to self-monitor for what adverse effect? A) Tinnitus B) Visual changes C) Stomatitis D) Hirsutism

B) Visual changes Antimalarials may cause visual changes; regular ophthalmologic examinations are necessary.

A nurse is reviewing a patients medication administration record in an effort to identify drugs that may contribute to the patients recent immunosuppression. What drug is most likely to have this effect? A) An antibiotic B) A nonsteroidal anti-inflammatory drug (NSAID) C) An antineoplastic D) An antiretroviral

C) An antineoplastic antineoplastic drugs have the most pronounced immunosuppressive effect.

A nurse is caring for a patient who is suspected of having giant cell arteritis (GCA). What laboratory tests are most useful in diagnosing this rheumatic disorder? Select all that apply. A) Erythrocyte count B) Erythrocyte sedimentation rate C) Creatinine clearance D) C-reactive protein E) D-dimer

B,D B) Erythrocyte sedimentation rate D) C-reactive protein Simultaneous elevation in the ESR and CRP have a sensitivity of 88% and a specificity of 98% in making the diagnosis of GCA when coupled with clinical findings.

Research has shown that patient adherence to prescribed regimens is generally low, especially when the patient will have to follow the regimen for a long time. Which of the following individuals would most likely benefit from health education that emphasizes adherence? A) An older adult who is colonized with methicillin-resistant Staphylococcus aureus (MRSA) B) An 80-year-old man who has a small bowel obstruction C) A 52-year-old woman who has a new diagnosis of multiple sclerosis D) A child who fractured her humerus in a playground accident

C) A 52-year-old woman who has a new diagnosis of multiple sclerosis Many people do not adhere to their prescribed regimens; rates of adherence are generally low, especially when the regimens are complex or of long duration (e.g., therapy for tuberculosis, multiple sclerosis, and HIV infection and hemodialysis).

A nurse is caring for a patient hospitalized with AIDS. A friend comes to visit the patient and privately asks the nurse about the risk of contracting HIV when visiting the patient. What is the nurses best response? A) Do you think that you might already have HIV? B) Dont worry. Your immune system is likely very healthy. C) AIDS isnt transmitted by casual contact. D) You cant contract AIDS in a hospital setting.

C) AIDS isnt transmitted by casual contact. AIDS is commonly transmitted by contact with blood and body fluids. Patients, family, and friends must be reassured that HIV is not spread through casual contact. A healthy immune system is not necessarily a protection against HIV. A hospital setting does not necessarily preclude HIV infection.

A patient requires ongoing treatment and infection-control precautions because of an inherited deficit in immune function. The nurse should recognize that this patient most likely has what type of immune disorder? A) A primary immune deficiency B) A gammopathy C) An autoimmune disorder D) A rheumatic disorder

C) An autoimmune disorder Autoimmune disorders are less likely to have a genetic component, though some have a genetic component.

A nurse is caring for a patient who has had a severe antigen/antibody reaction. The nurse knows that the portion of the antigen that is involved in binding with the antibody is called what? A) Antibody lock B) Antigenic sequence C) Antigenic determinant D) Antibody channel

C) Antigenic determinant The portion of the antigen involved in binding with the antibody is referred to as the antigenic determinant. This portion is not known as an antibody lock, antigenic sequence, or antibody channel.

A patient has informed the home health nurse that she has recently noticed distortions when she looks at the Amsler grid that she has mounted on her refrigerator. What is the nurses most appropriate action? A) Reassure the patient that this is an age-related change in vision. B) Arrange for the patient to have her visual acuity assessed. C) Arrange for the patient to be assessed for macular degeneration. D) Facilitate tonometry testing.

C) Arrange for the patient to be assessed for macular degeneration. The Amsler grid is a test often used for patients with macular problems, such as macular degeneration.

A patient with rheumatoid arthritis comes into the clinic for a routine check-up. On assessment the nurse notes that the patient appears to have lost some of her ability to function since her last office visit. Which of the following is the most appropriate action? A) Arrange a family meeting in order to explore assisted living options. B) Refer the patient to a support group. C) Arrange for the patient to be assessed in her home environment. D) Refer the patient to social work.

C) Arrange for the patient to be assessed in her home environment. Assessment in the patients home setting can often reveal more meaningful data than an assessment in the health care setting.

An older adult patient has been diagnosed with macular degeneration and the nurse is assessing him for changes in visual acuity since his last clinic visit. When assessing the patient for recent changes in visual acuity, the patient states that he sees the lines on an Amsler grid as being distorted. What is the nurses most appropriate response? A) Ask if the patient has been using OTC vasoconstrictors. B) Instruct the patient to repeat the test at different times of the day when at home. C) Arrange for the patient to visit his ophthalmologist. D) Encourage the patient to adhere to his prescribed drug regimen.

C) Arrange for the patient to visit his ophthalmologist. With a change in the patients perception of the grid, the patient should notify the ophthalmologist immediately and should arrange to be seen promptly.

A patient has just arrived to the floor after an enucleation procedure following a workplace accident in which his left eye was irreparably damaged. Which of the following should the nurse prioritize during the patients immediate postoperative recovery? A) Teaching the patient about options for eye prostheses B) Teaching the patient to estimate depth and distance with the use of one eye C) Assessing and addressing the patients emotional needs D) Teaching the patient about his post-discharge medication regimen

C) Assessing and addressing the patients emotional needs When surgical eye removal is unexpected, such as in severe ocular trauma, leaving no time for the patient and family to prepare for the loss, the nurses role in providing emotional support is crucial.

Diagnostic testing has revealed a deficiency in the function of a patients complement system. This patient is likely to have an impaired ability to do which of the following? A) Protecting the body against viral infection B) Marking the parameters of the immune response C) Bridging natural and acquired immunity D) Collecting immune complexes during inflammation

C) Bridging natural and acquired immunity Complement has three major physiologic functions: defending the body against bacterial infection, bridging natural and acquired immunity, and disposing of immune complexes and the byproducts associated with inflammation.

The mother of two young children has been diagnosed with HIV and expresses fear of dying. How should the nurse best respond to the patient? A) Would you like me to have the chaplain come speak with you? B) You'll learn much about the promise of a cure for HIV. C) Can you tell me what concerns you most about dying? D) You need to maintain hope because you may live for several years.

C) Can you tell me what concerns you most about dying? The nurse can help the patient verbalize feelings and identify resources for support. The nurse should respond with an open-ended question to help the patient to identify fears about being diagnosed with a life-threatening chronic illness.

A patients recent diagnostic testing included a total lymphocyte count. The results of this test will allow the care team to gauge what aspect of the patients immunity? A) Humoral immune function B) Antigen recognition C) Cell-mediated immune function D) Antibody production

C) Cell-mediated immune function A total lymphocyte count is a test used to determine cellular immune function. It is not normally used for testing humoral immune function and the associated antigenantibody.

A patient with cystic fibrosis has received a double lung transplant and is now experiencing signs of rejection. What is the immune response that predominates in this situation? A) Humoral B) Nonspecific C) Cellular D) Mitigated

C) Cellular Most immune responses to antigens involve both humoral and cellular responses, although only one predominates.

A patients current immune response involves the direct destruction of foreign microorganisms. This aspect of the immune response may be performed by what cells? A) Suppressor T cells B) Memory T cells C) Cytotoxic T cells D) Complement T cells

C) Cytotoxic T cells Cytotoxic T cells (also called CD8 + cells) participate in the destruction of foreign organisms.

A gerontologic nurse is caring for an older adult patient who has a diagnosis of pneumonia. What age- related change increases older adults susceptibility to respiratory infections? A) Atrophy of the thymus B) Bronchial stenosis C) Impaired ciliary action D) Decreased diaphragmatic muscle tone

C) Impaired ciliary action As a consequence of impaired ciliary action due to exposure to smoke and environmental toxins, older adults are vulnerable to lung infections.

You are the nurse planning to teach tracheostomy care to a patient who will be discharged home following a spinal cord injury. When preparing your teaching, which of the following is the most important component of your teaching plan? A) Citing the evidence that underlies each of your teaching points B) Alleviating the patients guilt associated with not knowing appropriate self-care C) Determining the patients readiness to learn new information D) Including your nursing colleagues in the planning process

C) Determining the patients readiness to learn new information Assessment in the teachinglearning process is directed toward the systematic collection of data about the person and familys learning needs and readiness to learn. Patient readiness is critical to accepting and integrating new information. Unless the patient is ready to accept new information, patient teaching will be ineffective.

A nurse is working with a teenage boy who was recently diagnosed with asthma. During the current session, the nurse has taught the boy how to administer his bronchodilator by metered-dose inhaler. How should the nurse evaluate the teaching learning process? A) Ask the boy specific questions about his medication. B) Ask the boy whether he now understands how to use his inhaler. C) Directly observe the boy using his inhaler to give himself a dose. D) Assess the boys respiratory health at the next scheduled visit.

C) Directly observe the boy using his inhaler to give himself a dose. Demonstration and practice are essential ingredients of a teaching program, especially when teaching skills. It is best to demonstrate the skill and then give the learner ample opportunity for practice. By observing the patient using the inhaler, the nurse may identify what learning needs to be enhanced or reinforced.

The nursing instructor has given an assignment to a group of certified nurse practitioner (CNP) students. They are to break into groups of four and complete a health-promotion teaching project and present a report to their fellow students. What project most clearly demonstrates the principles of health- promotion teaching? A) Demonstrating an injection technique to a patient for anticoagulant therapy B) Explaining the side effects of a medication to an adult patient C) Discussing the importance of preventing sexually transmitted infections (STI) to a group of high school students D) Instructing an adolescent patient about safe and nutritious food preparation

C) Discussing the importance of preventing sexually transmitted infections (STI) to a group of high school students Health promotion encourages people to live a healthy lifestyle and to achieve a high level of wellness. Discussing the importance of STI prevention to a group of high school students is the best example of a health-promotion teaching project. This proactive intervention is a more precise example of health promotion than the other cited examples.

A clinic nurse is caring for a patient diagnosed with rheumatoid arthritis (RA). The patient tells the nurse that she has not been taking her medication because she usually cannot remove the childproof medication lids. How can the nurse best facilitate the patients adherence to her medication regimen? A) Encourage the patient to store the bottles with their tops removed. B) Have a trusted family member take over the management of the patients medication regimen. C) Encourage her to have her pharmacy replace the tops with alternatives that are easier to open. D) Have the patient approach her primary care provider to explore medication alternatives.

C) Encourage her to have her pharmacy replace the tops with alternatives that are easier to open. The patients pharmacy will likely be able to facilitate a practical solution that preserves the patients independence while still fostering adherence to treatment.

Cytomegalovirus (CMV) is the most common cause of retinal inflammation in patients with AIDS. What drug, surgically implanted, is used for the acute stage of CMV retinitis? A) Pilocarpine B) Penicillin C) Ganciclovir D) Gentamicin

C) Ganciclovir The surgically implanted sustained-release insert of ganciclovir enables higher concentrations of ganciclovir to reach the CMV retinitis.

The nurse knows that the response of natural immunity is enhanced by processes that are inherent in the physical and chemical barriers of the body. What is a chemical barrier that enhances the response of natural immunity? A) Cell cytoplasm B) Interstitial fluid C) Gastric secretions D) Cerebrospinal fluid

C) Gastric secretions Chemical barriers, such as mucus, acidic gastric secretions, enzymes in tears and saliva, and substances in sebaceous and sweat secretions, act in a nonspecific way to destroy invading bacteria and fungi. Not all body fluids are chemical barriers, however. Cell cytoplasm, interstitial fluid, and CSF are not normally categorized as chemical barriers to infection.

A 6-month-old infant is brought to the ED by his parents for inconsolable crying and pulling at his right ear. When assessing this infant, the advanced practice nurse is aware that the tympanic membrane should be what color in a healthy ear? A) Yellowish-white B) Pink C) Gray D) Bluish-white

C) Gray The healthy tympanic membrane appears pearly gray and is positioned obliquely at the base of the ear canal. Any other color is suggestive of a pathological process.

Positive patient outcomes are the ultimate goal of nursepatient interactions, regardless of the particular setting. Which of the following factors has the most direct influence on positive patient-care outcomes? A) Patients age B) Patients ethnic heritage C) Health education D) Outcome evaluation

C) Health education Health education is an influential factor directly related to positive patient-care outcomes.

A nursing student is collaborating with a public health nurse on a local health promotion initiative and they recognize the need for a common understanding of health. How should the student and the nurse best define health? A) Health is an outcome systematically maximizing wellness. B) Health is a state that is characterized by a lack of disease. C) Health is a condition that enables people to function at their optimal potential. D) Health is deliberate attempt to mitigate the effects of disease.

C) Health is a condition that enables people to function at their optimal potential. Health is viewed as a dynamic, ever-changing condition that enables people to function at an optimal potential at any given time.

A patient with systemic lupus erythematosus (SLE) is preparing for discharge. The nurse knows that the patient has understood health education when the patient makes what statement? A) Ill make sure I get enough exposure to sunlight to keep up my vitamin D levels. B) Ill try to be as physically active as possible between flare-ups. C) Ill make sure to monitor my body temperature on a regular basis. D) Ill stop taking my steroids when I get relief from my symptoms.

C) Ill make sure to monitor my body temperature on a regular basis. Fever can signal an exacerbation and should be reported to the physician. Sunlight and other sources of ultraviolet light may precipitate severe skin reactions and exacerbate the disease.

A clinic nurse is caring for a patient with suspected gout. While explaining the pathophysiology of gout to the patient, the nurse should describe which of the following? A) Autoimmune processes in the joints B) Chronic metabolic acidosis C) Increased uric acid levels D) Unstable serum calcium levels

C) Increased uric acid levels Gout is caused by hyperuricemia (increased serum uric acid). Gout is not categorized as an autoimmune disease and it does not result from metabolic acidosis or unstable serum calcium levels.

The nurse is working with a male patient who has diagnoses of coronary artery disease and angina pectoris. During a clinic visit, the nurse learns that he has only been taking his prescribed antiplatelet medication when he experiences chest pain and fatigue. What nursing diagnosis is most relevant to this assessment finding? A) Acute pain related to myocardial ischemia B) Confusion related to mismanagement of drug regimen C) Ineffective health maintenance related to inappropriate medication use D) Ineffective role performance related to inability to manage medications

C) Ineffective health maintenance related to inappropriate medication use This patients actions suggest that by taking his medications incorrectly he is not adequately maintaining his health.

A patient with SLE has come to the clinic for a routine check-up. When auscultating the patients apical heart rate, the nurse notes the presence of a distinct scratching sound. What is the nurses most appropriate action? A) Reposition the patient and auscultate posteriorly. B) Document the presence of S3 and monitor the patient closely. C) Inform the primary care provider that a friction rub may be present. D) Inform the primary care provider that the patient may have pneumonia.

C) Inform the primary care provider that a friction rub may be present. Patients with SLE are susceptible to developing a pericardial friction rub, possibly associated with myocarditis and accompanying pleural effusions; this warrants prompt medical follow-up.

A nurse is performing the health history and physical assessment of a patient who has a diagnosis of rheumatoid arthritis (RA). What assessment finding is most consistent with the clinical presentation of RA? A) Cool joints with decreased range of motion B) Signs of systemic infection C) Joint stiffness, especially in the morning D) Visible atrophy of the knee and shoulder joints

C) Joint stiffness, especially in the morning In addition to joint pain and swelling, another classic sign of RA is joint stiffness, especially in the morning. Joints are typically swollen, not atrophied, and systemic infection does not accompany the disease. Joints are often warm rather than cool.

The nurse care plan for a patient with AIDS includes the diagnosis of Risk for Impaired Skin Integrity. What nursing intervention should be included in the plan of care? A) Maximize the patients fluid intake. B) Provide total parenteral nutrition (TPN). C) Keep the patients bed linens free of wrinkles. D) Provide the patient with snug clothing at all times.

C) Keep the patients bed linens free of wrinkles. Skin surfaces are protected from friction and rubbing by keeping bed linens free of wrinkles and avoiding tight or restrictive clothing.

A nurse is educating a patient with gout about lifestyle modifications that can help control the signs and symptoms of the disease. What recommendation should the nurse make? A) Ensuring adequate rest B) Limiting exposure to sunlight C) Limiting intake of alcohol D) Smoking cessation

C) Limiting intake of alcohol Alcohol and red meat can precipitate an acute exacerbation of gout.

A patient with a recent diagnosis of HIV infection expresses an interest in exploring alternative and complementary therapies. How should the nurse best respond? A) Complementary therapies generally have not been approved, so patients are usually discouraged from using them. B) Researchers have not looked at the benefits of alternative therapy for patients with HIV, so we suggest that you stay away from these therapies until there is solid research data available. C) Many patients with HIV use some type of alternative therapy and, as with most health treatments, there are benefits and risks. D) You'll need to meet with your doctor to choose between an alternative approach to treatment and a medical approach.

C) Many patients with HIV use some type of alternative therapy and, as with most health treatments, there are benefits and risks. The nurse should approach the topic of alternative or complementary therapies from an open-ended, supportive approach, emphasizing the need to communicate with care providers.

A patient with AIDS is admitted to the hospital with AIDS-related wasting syndrome and AIDS-related anorexia. What drug has been found to promote significant weight gain in AIDS patients by increasing body fat stores? A) Advera B) Momordicacharantia C) Megestrol D) Ranitidine

C) Megestrol Megestrol acetate (Megace), a synthetic oral progesterone preparation, promotes significant weight gain. In patients with HIV infection, it increases body weight primarily by increasing body fat stores.

The nurse is assessing a patient with multiple sclerosis who is demonstrating involuntary, rhythmic eye movements. What term will the nurse use when documenting these eye movements? A) Vertigo B) Tinnitus C) Nystagmus D) Astigmatism

C) Nystagmus Nystagmus refers to involuntary rhythmic eye movement.

When administering a patients eye drops, the nurse recognizes the need to prevent absorption by the nasolacrimal duct. How can the nurse best achieve this goal? A) Ensure that the patient is well hydrated at all times. B) Encourage self-administration of eye drops. C) Occlude the puncta after applying the medication. D) Position the patient supine before administering eye drops.

C) Occlude the puncta after applying the medication. Absorption of eye drops by the nasolacrimal duct is undesirable because of the potential systemic side effects of ocular medications. To diminish systemic absorption and minimize the side effects, it is important to occlude the puncta.

The clinic nurse is assessing a child who has been brought to the clinic with signs and symptoms that are suggestive of otitis externa. What assessment finding is characteristic of otitis externa? A) Tophi on the pinna and ear lobe B) Dark yellow cerumen in the external auditory canal C) Pain on manipulation of the auricle D) Air bubbles visible in the middle ear

C) Pain on manipulation of the auricle Pain when the nurse pulls gently on the auricle in preparation for an otoscopic examination of the ear canal is a characteristic finding in patients with otitis externa.

The nurse is caring for a patient who has been admitted for the treatment of AIDS. In the morning, the patient tells the nurse that he experienced night sweats and recently coughed up some blood. What is the nurses most appropriate action? A) Assess the patient for additional signs and symptoms of Kaposis sarcoma. B) Review the patients most recent viral load and CD4+ count. C) Place the patient on respiratory isolation and inform the physician. D)Perform oral suctioning to reduce the patients risk for aspiration.

C) Place the patient on respiratory isolation and inform the physician. These signs and symptoms are suggestive of tuberculosis, not Kaposis sarcoma; prompt assessment and treatment is necessary.

A nurse is admitting a patient who exhibits signs and symptoms of a nutritional deficit. Inadequate intake of what nutrient increases a patients susceptibility to infection? A) Vitamin B12 B) Unsaturated fats C) Proteins D) Complex carbohydrates

C) Proteins Depletion of protein reserves results in atrophy of lymphoid tissues, depression of antibody response, reduction in the number of circulating T cells, and impaired phagocytic function. As a result, the patient has an increased susceptibility to infection.

Which of the following nursing interventions would most likely facilitate effective communication with a hearing-impaired patient? A) Ask the patient to repeat what was said in order to evaluate understanding. B) Stand directly in front of the patient to facilitate lip reading. C) Reduce environmental noise and distractions before communicating. D) Raise the voice to project sound at a higher frequency.

C) Reduce environmental noise and distractions before communicating. Communication with the hearing impaired can be facilitated by talking in a quiet space free of competing noise stimuli and other distractions.

The nurse is planning the care of a patient with a diagnosis of vertigo. What nursing diagnosis risk should the nurse prioritize in this patients care? A) Risk for disturbed sensory perception B) Risk for unilateral neglect C) Risk for falls D) Risk for ineffective health maintenance

C) Risk for falls Vertigo is defined as the misperception or illusion of motion, either of the person or the surroundings. A patient suffering from vertigo will be at an increased risk of falls.

A patient is being discharged home after mastoid surgery. What topic should the nurse address in the patients discharge education? A) Expected changes in facial nerve function B) The need for audiometry testing every 6 months following recovery C) Safe use of analgesics and antivertiginous agents D) Appropriate use of OTC ear drops

C) Safe use of analgesics and antivertiginous agents Patients require instruction about medication therapy, such as analgesics and antivertiginous agents (e.g., antihistamines) prescribed for balance disturbance.

A patient has been diagnosed with hearing loss related to damage of the end organ for hearing or cranial nerve VIII. What term is used to describe this condition? A) Exostoses B) Otalgia C) Sensorineural hearing loss D) Presbycusis

C) Sensorineural hearing loss Sensorineural hearing loss is loss of hearing related to damage of the end organ for hearing or cranial nerve VIII.

A patient with SLE asks the nurse why she has to come to the office so often for check-ups. What would be the nurses best response? A) Taking care of you in the best way involves seeing you face to face. B) Taking care of you in the best way involves making sure you are taking your medication the way it is ordered. C) Taking care of you in the best way involves monitoring your disease activity and how well the prescribed treatment is working. D) Taking care of you in the best way involves drawing blood work every month.

C) Taking care of you in the best way involves monitoring your disease activity and how well the prescribed treatment is working. The goals of treatment include preventing progressive loss of organ function, reducing the likelihood of acute disease, minimizing disease-related disabilities, and preventing complications from therapy.

The nurse is teaching a patient to care for her new ocular prosthesis. What should the nurse emphasize during the patients health education? A) The need to limit exposure to bright light B) The need to maintain a low Fowlers position when removing the prosthesis C) The need to perform thorough hand hygiene before handling the prosthesis D) The need to apply antiviral ointment to the prosthesis daily

C) The need to perform thorough hand hygiene before handling the prosthesis Proper hand hygiene must be observed before inserting and removing an ocular prosthesis.

A nurse would identify that a colleague needs additional instruction on standard precautions when the colleague exhibits which of the following behaviors? A) The nurse wears face protection, gloves, and a gown when irrigating a wound. B) The nurse washes the hands with a waterless antiseptic agent after removing a pair of soiled gloves. C) The nurse puts on a second pair of gloves over soiled gloves while performing a bloody procedure. D) The nurse places a used needle and syringe in the puncture-resistant container without capping the needle.

C) The nurse puts on a second pair of gloves over soiled gloves while performing a bloody procedure. Gloves must be changed after contact with materials that may contain high concentration of microorganisms, even when working with the same patient. Each of the other listed actions adheres to standard precautions.

A patient with low vision has called the clinic and asked the nurse for help with acquiring some low- vision aids. What else can the nurse offer to help this patient manage his low vision? A) The patient uses OTC NSAIDs. B) The patient has a history of stroke. C) The patient has diabetes. D) The patient has Asian ancestry.

C) The patient has diabetes. Diabetes is a risk factor for glaucoma, but Asian ancestry, NSAIDs, and stroke are not risk factors for the disease.

A patient has been diagnosed with serous otitis media for the third time in the past year. How should the nurse best interpret this patients health status? A) For some patients, these recurrent infections constitute an age-related physiologic change. B) The patient would benefit from a temporary mobility restriction to facilitate healing. C) The patient needs to be assessed for nasopharyngeal cancer. D) Blood cultures should be drawn to rule out a systemic infection.

C) The patient needs to be assessed for nasopharyngeal cancer. A carcinoma (e.g., nasopharyngeal cancer) obstructing the eustachian tube should be ruled out in adults with persistent unilateral serous otitis media.

A patient is responding to a microbial invasion and the patients differentiated lymphocytes have begun to function in either a humoral or a cellular capacity. During what stage of the immune response does this occur? A) The recognition stage B) The effector stage C) The response stage D) The proliferation stage

C) The response stage In the response stage, the differentiated lymphocytes function in either a humoral or a cellular capacity.

A 56-year-old patient has come to the clinic for his routine eye examination and is told he needs bifocals. The patient asks the nurse what change in his eyes has caused his need for bifocals. How should the nurse respond? A) You know, you are getting older now and we change as we get older. B) The parts of our eyes age, just like the rest of us, and this is nothing to cause you to worry. C) There is a gradual thickening of the lens of the eye and it can limit the eyes ability for accommodation. D) The eye gets shorter, back to front, as we age and it changes how we see things.

C) There is a gradual thickening of the lens of the eye and it can limit the eyes ability for accommodation. As a result of a loss of accommodative power in the lens with age, many adults require bifocals or other forms of visual correction. This is not attributable to a change in the shape of the ocular globe. The nurse should not dismiss or downplay the patients concerns.

A recent nursing graduate is aware that the nursing scope of practice goes far beyond what is characterized as bedside care. Which of the following is a nurses primary responsibility? A) To promote activities that enhance community cohesion B) To encourage individuals self-awareness C) To promote activities that foster well-being D) To influence individuals social interactions

C) To promote activities that foster well-being As health care professionals, nurses have a responsibility to promote activities that foster well-being, self-actualization, and personal fulfillment.

A patients primary infection with HIV has subsided and an equilibrium now exists between HIV levels and the patients immune response. This physiologic state is known as which of the following? A) Static stage B) Latent stage C) Viral set point D) Window period

C) Viral set point The remaining amount of virus in the body after primary infection is referred to as the viral set point, which results in a steady state of infection that lasts for years.

A patient was tested for HIV using enzyme immunoassay (EIA) and results were positive. The nurse should expect the primary care provider to order what test to confirm the EIA test results? A) Another EIA test B) Viral load test C) Western blot test D) CD4/CD8 ratio

C) Western blot test The Western blot test detects antibodies to HIV and is used to confirm the EIA test results.

A child goes to the school nurse and complains of not being able to hear the teacher. What test could the school nurse perform that would preliminarily indicate hearing loss? A) Audiometry B) Rinne test C) Whisper test D) Weber test

C) Whisper test A general estimate of hearing can be made by assessing the patients ability to hear a whispered phrase or a ticking watch, testing one ear at a time.

A nurse is planning the assessment of a patient who is exhibiting signs and symptoms of an autoimmune disorder. The nurse should be aware that the incidence and prevalence of autoimmune diseases is known to be higher among what group? A) Young adults B) Native Americans C) Women D) Hispanics

C) Women Many autoimmune diseases have a higher incidence in females than in males, a phenomenon believed to be correlated with sex hormones.

A nurse is creating a teaching plan for a patient who has a recent diagnosis of scleroderma. What topics should the nurse address during health education? Select all that apply. A) Surgical treatment options B) The importance of weight loss C) Managing Raynauds-type symptoms D) Smoking cessation E) The importance of vigilant skin care

C,D,E C) Managing Raynauds-type symptoms D) Smoking cessation E) The importance of vigilant skin care Patient teaching for the patient with scleroderma focuses on management of Raynauds phenomenon, smoking cessation, and meticulous skin care.

An older adult with a recent history of mixed hearing loss has been diagnosed with a cholesteatoma. What should this patient be taught about this diagnosis? Select all that apply A) Cholesteatomas are benign and self-limiting, and hearing loss will resolve spontaneously. B) Cholesteatomas are usually the result of metastasis from a distant tumor site. C) Cholesteatomas are often the result of chronic otitis media. D) Cholesteatomas, if left untreated, result in intractable neuropathic pain. E) Cholesteatomas usually must be removed surgically.

C,E C) Cholesteatomas are often the result of chronic otitis media. E) Cholesteatomas usually must be removed surgically. Cholesteatoma is a tumor of the external layer of the eardrum into the middle ear, often resulting from chronic otitis media.

A clinic nurse is caring for a patient newly diagnosed with fibromyalgia. When developing a care plan for this patient, what would be a priority nursing diagnosis for this patient? A) Impaired Urinary Elimination Related to Neuropathy B) Altered Nutrition Related to Impaired Absorption C) Disturbed Sleep Pattern Related to CNS Stimulation D) Fatigue Related to Pain

D) Fatigue Related to Pain Fibromyalgia is characterized by fatigue, generalized muscle aching, and stiffness.

On otoscopy, a red blemish behind the tympanic membrane is suggestive of what diagnosis? A) Acoustic tumor B) Cholesteatoma C) Facial nerve neuroma D) Glomus tympanicum

D) Glomus tympanicum In the case of glomus tympanicum, a red blemish on or behind the tympanic membrane is seen on otoscopy.

The nurse is administering eye drops to a patient with glaucoma. After instilling the patients first medication, how long should the nurse wait before instilling the patients second medication into the same eye? A) 30 seconds B) 1 minute C) 3 minutes D) 5 minutes

D) 5 minutes A 5-minute interval between successive eye drop administrations allows for adequate drug retention and absorption. Any time frame less than 5 minutes will not allow adequate absorption.

The nurse is preparing discharge teaching for a 51-year-old woman diagnosed with urinary retention secondary to multiple sclerosis. The nurse will teach the patient to self-catheterize at home upon discharge. What teaching method is most likely to be effective for this patient? A) A list of clear instructions written at a sixth-grade level B) A short video providing useful information and demonstrations C) An audio-recorded version of discharge instructions that can be accessed at home D) A discussion and demonstration between the nurse and the patient

D) A discussion and demonstration between the nurse and the patient Demonstration and practice are essential ingredients of a teaching program, especially when teaching skills. It is best to demonstrate the skill and then give the learner ample opportunity for practice.

A patient is being discharged home from the ambulatory surgical center after cataract surgery. In reviewing the discharge instructions with the patient, the nurse instructs the patient to immediately call the office if the patient experiences what? A) Slight morning discharge from the eye B) Any appearance of redness of the eye C) A scratchy feeling in the eye D) A new floater in vision

D) A new floater in vision Cataract surgery increases the risk of retinal detachment and the patient must be instructed to notify the surgeon of new floaters in vision, flashing lights, decrease in vision, pain, or increase in redness.

A nurse is reviewing the immune system before planning an immunocompromised patients care. How should the nurse characterize the humoral immune response? A) Specialized cells recognize and ingest cells that are recognized as foreign. B) T lymphocytes are assisted by cytokines to fight infection. C) Lymphocytesare stimulated to become cells that attack microbes directly. D) Antibodies are made by B lymphocytes in response to a specific antigen.

D) Antibodies are made by B lymphocytes in response to a specific antigen. The humoral response is characterized by the production of antibodies by B lymphocytes in response to a specific antigen. Phagocytosis and direct attack on microbes occur in the context of the cellular immune response.

A patient is suspected of having rheumatoid arthritis and her diagnostic regimen includes aspiration of synovial fluid from the knee for a definitive diagnosis. The nurse knows that which of the following procedures will be involved? A) Angiography B) Myelography C) Paracentesis D) Arthocentesis

D) Arthocentesis Arthrocentesis involves needle aspiration of synovial fluid. Angiography is an x-ray study of circulation with a contrast agent injected into a selected artery.

A nurse is providing care for a patient who has a recent diagnosis of giant cell arteritis (GCA). What aspect of physical assessment should the nurse prioritize? A) Assessment for subtle signs of bleeding disorders B) Assessment of the metatarsal joints and phalangeal joints C) Assessment for thoracic pain that is exacerbated by activity D) Assessment for headaches and jaw pain

D) Assessment for headaches and jaw pain Assessment of the patient with GCA focuses on musculoskeletal tenderness, weakness, and decreased function. Careful attention should be directed toward assessing the head (for changes in vision, headaches, and jaw claudication).

After mastoid surgery, an 81-year-old patient has been identified as needing assistance in her home. What would be a primary focus of this patients home care? A) Preparation of nutritious meals and avoidance of contraindicated foods B) Ensuring the patient receives adequate rest each day C) Helping the patient adapt to temporary hearing loss D) Assisting the patient with ambulation as needed to avoid falling

D) Assisting the patient with ambulation as needed to avoid falling The caregiver and patient are cautioned that the patient may experience some vertigo and will therefore require help with ambulation to avoid falling.

A nurse is teaching preventative measures for otitis externa to a group of older adults. What action should the nurse encourage? A) Rinsing the ears with normal saline after swimming B) Avoiding loud environmental noises C) Instilling antibiotic ointments on a regular basis D) Avoiding the use of cotton swabs

D) Avoiding the use of cotton swabs Nurses should instruct patients not to clean the external auditory canal with cotton-tipped applicators and to avoid events that traumatize the external canal such as scratching the canal with the fingernail or other objects.

Which of the following nurses actions carries the greatest potential to prevent hearing loss due to ototoxicity? A) Ensure that patients understand the differences between sensory hearing loss and conductive hearing loss. B) Educate patients about expected age-related changes in hearing perception. C) Educate patients about the risks associated with prolonged exposure to environmental noise. D) Be aware of patients medication regimens and collaborate with other professionals accordingly.

D) Be aware of patients medication regimens and collaborate with other professionals accordingly. A variety of medications may have adverse effects on the cochlea, vestibular apparatus, or cranial nerve VIII. All but a few, such as aspirin and quinine, cause irreversible hearing loss.

A public health nurse understands that health promotion should continue across the lifespan. When planning health promotion initiatives, when in the lifespan should health promotion begin? A) Adolescence B) School age C) Preschool D) Before birth

D) Before birth Health promotion should begin prior to birth because the health practices of a mother prior to the birth of her child can be influenced positively or negatively. This makes the other options incorrect.

An adult patient will be receiving outpatient intravenous antibiotic therapy for the treatment of endocarditis. The nurse is preparing to perform health education to ensure the patients adherence to the course of treatment. Which of the following assessments should be the nurses immediate priority? A) Patients understanding of the teaching plan B) Quality of the patients relationships C) Patients previous medical history D) Characteristics of the patients culture

D) Characteristics of the patients culture Before beginning health teaching, nurses must conduct an individual cultural assessment instead of relying only on generalized assumptions about a particular culture.

Following a motorcycle accident, a 17-year-old man is brought to the ED. What physical assessment findings related to the ear should be reported by the nurse immediately? A) The malleus can be visualized during otoscopic examination. B) The tympanic membrane is pearly gray. C) Tenderness is reported by the patient when the mastoid area is palpated. D) Clear, watery fluid is draining from the patients ear.

D) Clear, watery fluid is draining from the patients ear. For the patient experiencing acute head trauma, immediately report the presence of clear, watery drainage from the ear.

Allopurinol (Zyloprim) has been ordered for a patient receiving treatment for gout. The nurse caring for this patient knows to assess the patient for bone marrow suppression, which may be manifested by which of the following diagnostic findings? A) Hyperuricemia B) Increased erythrocyte sedimentation rate C) Elevated serum creatinine D) Decreased platelets

D) Decreased platelets Thrombocytopenia occurs in bone marrow suppression.

A patient is being treated for cancer and the nurse has identified the nursing diagnosis of Risk for Infection Due to Protein Losses. Protein losses inhibit immune response in which of the following ways? A) Causing apoptosis of cytokines B) Increasing interferon production C) Causing CD4+ cells to mutate D) Depressing antibody response

D) Depressing antibody response Depletion of protein reserves results in atrophy of lymphoid tissues, depression of antibody response, reduction in the number of circulating T cells, and impaired phagocytic function.

A nursing student is giving a report on the immune system. What function of cytokines should the student describe? A) Determining whether a cell is foreign B) Determining if lymphokines will be activated C) Determining whether the T cells will remain in the nodes and retain a memory of the antigen D) Determining whether the immune response will be the production of antibodies or a cell-mediated response

D) Determining whether the immune response will be the production of antibodies or a cell-mediated response Separate subpopulations of helper T cells produce different types of cytokines and determine whether the immune response will be the production of antibodies or a cell-mediated immune response.

The nurse is discharging a patient home after mastoid surgery. What should the nurse include in discharge teaching? A) Try to induce a sneeze every 4 hours to equalize pressure. B) Be sure to exercise to reduce fatigue. C) Avoid sleeping in a side-lying position. D) Don't blow your nose for 2 to 3 weeks.

D) Don't blow your nose for 2 to 3 weeks. The patient is instructed to avoid heavy lifting, straining, exertion, and nose blowing for 2 to 3 weeks after surgery to prevent dislodging the tympanic membrane graft or ossicular prosthesis. Side-lying is not contraindicated; sneezing could cause trauma.

A nurse is planning a patients care and is relating it to normal immune response. During what stage of the immune response should the nurse know that antibodies or cytotoxic T cells combine and destroy the invading microbes? A) Recognition stage B) Proliferation stage C) Response stage D) Effector stage

D) Effector stage In the effector stage, either the antibody of the humoral response or the cytotoxic (killer) T cell of the cellular response reaches and couples with the antigen on the surface of the foreign invader.

A patient is being treated for bacterial pneumonia. In the first stages of illness, the patients dyspnea was accompanied by a high fever. Currently, the patient claims to be feeling better and is afebrile. The patient is most likely in which stage of the immune response? A) Recognition stage B) Proliferation stage C) Response stage D) Effector stage

D) Effector stage The immune response culminates with the effector stage, during which offending microorganisms are killed by the various actions of the immune system. The patients improvement in health status is likely the result of this final stage in the immune response.

A nurse is assessing a patient with rheumatoid arthritis. The patient expresses his intent to pursue complementary and alternative therapies. What fact should underlie the nurses response to the patient? A) New evidence shows CAM to be as effective as medical treatment. B) CAM therapies negate many of the benefits of medications. C) CAM therapies typically do more harm than good. D) Evidence shows minimal benefits from most CAM therapies.

D) Evidence shows minimal benefits from most CAM therapies. A recent systematic review of complementary and alternative medicine (CAM) examined the efficacy of herbal medicine, acupuncture, Tai chi and biofeedback for the treatment of rheumatoid arthritis and osteoarthritis.

The nurse is addressing condom use in the context of a health promotion workshop. When discussing the correct use of condoms, what should the nurse tell the attendees? A) Attach the condom prior to erection. B) A condom may be reused with the same partner if ejaculation has not occurred. C) Use skin lotion as a lubricant if alternatives are unavailable. D) Hold the condom by the cuff upon withdrawal.

D) Hold the condom by the cuff upon withdrawal. The condom should be unrolled over the hard penis before any kind of sex. The condom should be held by the tip to squeeze out air.

A nurse has admitted a patient who has been diagnosed with urosepsis. What immune response predominates in sepsis? A) Mitigated B) Nonspecific C) Cellular D) Humoral

D) Humoral Most immune responses to antigens involve both humoral and cellular responses, although only one predominates. For example, during transplantation rejection, the cellular response predominates, whereas in the bacterial pneumonias and sepsis, the humoral response plays the dominant role.

A patient is ready to be discharged home after a cataract extraction with intraocular lens implant and the nurse is reviewing signs and symptoms that need to be reported to the ophthalmologist immediately. Which of the patients statements best demonstrates an adequate understanding? A) I need to call the doctor if I get nauseated. B) I need to call the doctor if I have a light morning discharge. C) I need to call the doctor if I get a scratchy feeling. D) I need to call the doctor if I see flashing lights.

D) I need to call the doctor if I see flashing lights. Postoperatively, the patient who has undergone cataract extraction with intraocular lens implant should report new floaters in vision, flashing lights, decrease in vision, pain, or increase in redness to the ophthalmologist.

The public health nurse is addressing eye health and vision protection during an educational event. What statement by a participant best demonstrates an understanding of threats to vision? A) Im planning to avoid exposure to direct sunlight on my next vacation. B) Ive never exercised regularly, but Im going to start working out at the gym daily. C) Im planning to talk with my pharmacist to review my current medications. D) Im certainly going to keep a close eye on my blood pressure from now on.

D) Im certainly going to keep a close eye on my blood pressure from now on. Hypertension is a major cause of vision loss, exceeding the significance of inactivity, sunlight, and adverse effects of medications.

A child has been experiencing recurrent episodes of acute otitis media (AOM). The nurse should anticipate that what intervention is likely to be ordered? A) Ossiculoplasty B) Insertion of a cochlear implant C) Stapedectomy D) Insertion of a ventilation tube

D) Insertion of a ventilation tube If AOM recurs and there is no contraindication, a ventilating, or pressure-equalizing, tube may be inserted. The ventilating tube, which temporarily takes the place of the eustachian tube in equalizing pressure, is retained for 6 to 18 months.

Several residents of a long-term care facility have developed signs and symptoms of viral conjunctivitis. What is the most appropriate action of the nurse who oversees care in the facility? A) Arrange for the administration of prophylactic antibiotics to unaffected residents. B) Instill normal saline into the eyes of affected residents two to three times daily. C) Swab the conjunctiva of unaffected residents for culture and sensitivity testing. D) Isolate affected residents from residents who have not developed conjunctivitis.

D) Isolate affected residents from residents who have not developed conjunctivitis. To prevent spread during outbreaks of conjunctivitis caused by adenovirus, health care facilities must set aside specified areas for treating patients diagnosed with or suspected of having conjunctivitis caused by adenovirus.

An 18-year-old pregnant female has tested positive for HIV and asks the nurse if her baby is going to be born with HIV. What is the nurses best response? A) There is no way to know that for certain, but we do know that your baby has a one in four chance of being born with HIV. B) Your physician is likely the best one to ask that question. C) If the baby is HIV positive there is nothing that can be done until it is born, so try your best not to worry about it now. D) Its possible that your baby could contract HIV, either before, during, or after delivery.

D) Its possible that your baby could contract HIV, either before, during, or after delivery. Mother-to-child transmission of HIV-1 is possible and may occur in utero, at the time of delivery, or through breast-feeding.

A patient presents to the ED complaining of a sudden onset of incapacitating vertigo, with nausea and vomiting and tinnitus. The patient mentions to the nurse that she suddenly cannot hear very well. What would the nurse suspect the patients diagnosis will be? A) Ossiculitis B) Mnires disease C) Ototoxicity D) Labyrinthitis

D) Labyrinthitis Labyrinthitis is characterized by a sudden onset of incapacitating vertigo, usually with nausea and vomiting, various degrees of hearing loss, and possibly tinnitus.

The nurse is planning to teach a 75-year-old patient with coronary artery disease about administering her prescribed antiplatelet medication. How can the nurse best enhance the patients ability to learn? A) Provide links to Web sites that contain evidence-based information. B) Exclude family members from the session to prevent distraction. C) Use color-coded materials that are succinct and engaging. D) Make the information directly relevant to the patients condition.

D) Make the information directly relevant to the patients condition. Studies have shown that older adults can learn and remember if the information is paced appropriately, relevant, and followed by appropriate feedback.

A nurse has been working with Mrs. Griffin, a 71-year-old patient whose poorly controlled type 1 diabetes has led to numerous health problems. Over the past several years Mrs. Griffin has had several admissions to the hospital medical unit, and the nurse has often carried out health promotion interventions. Who is ultimately responsible for maintaining and promoting Mrs. Griffins health? A) The medical nurse B) The community health nurse who has also worked with Mrs. Griffin C) Mrs. Griffins primary care provider D) Mrs. Griffin

D) Mrs. Griffin American society places a great importance on health and the responsibility that each of us has to maintain and promote our own health. Therefore, the other options are incorrect.

A patients injury has initiated an immune response that involves inflammation. What are the first cells to arrive at a site of inflammation? A) Eosinophils B) Red blood cells C) Lymphocytes D) Neutrophils

D) Neutrophils Neutrophils are the first cells to arrive at the site where inflammation occurs.

The nurse is reviewing the health history of a newly admitted patient and reads that the patient has been previously diagnosed with exostoses. How should the nurse accommodate this fact into the patients plan of care? A) The nurse should perform the Rinne and Weber tests. B) The nurse should arrange for audiometry testing as soon as possible. C) The nurse should collaborate with the pharmacist to assess for potential ototoxic medications. D) No specific assessments or interventions are necessary to addressing exostoses.

D) No specific assessments or interventions are necessary to addressing exostoses. Exostoses are small, hard, bony protrusions found in the lower posterior bony portion of the ear canal; they usually occur bilaterally.

A neonate exhibited some preliminary signs of infection, but the infants condition resolved spontaneously prior to discharge home from the hospital. This infants recovery was most likely due to what type of immunity? A) Cytokine immunity B) Specific immunity C) Active acquired immunity D) Nonspecific immunity

D) Nonspecific immunity Natural immunity, or nonspecific immunity, is present at birth. Active acquired or specific immunity develops after birth. Cytokines are proteins that mediate the immune response; they are not a type of immunity

The nursing profession and nurses as individuals have a responsibility to promote activities that foster well-being. What factor has most influenced nurses abilities to play this vital role? A) Nurses are seen as nurturing professionals. B) Nurses possess a baccalaureate degree as the entry to practice. C) Nurses possess an authentic desire to help others. D) Nurses have long-established credibility with the public.

D) Nurses have long-established credibility with the public. Nurses, by virtue of their expertise in health and health care and their long-established credibility with consumers, play a vital role in health promotion.

A patient has been diagnosed with AIDS complicated by chronic diarrhea. What nursing intervention would be appropriate for this patient? A) Position the patient in the high Fowlers position whenever possible. B) Temporarily eliminate animal protein from the patients diet. C) Make sure the patient eats at least two servings of raw fruit each day. D) Obtain a stool culture to identify possible pathogens.

D) Obtain a stool culture to identify possible pathogens. A stool culture should be obtained to determine the possible presence of microorganisms that cause diarrhea.

A 16-year-old has come to the clinic and asks to talk to a nurse. The nurse asks the teen what she needs and the teen responds that she has become sexually active and is concerned about getting HIV. The teen asks the nurse what she can do keep from getting HIV. What would be the nurses best response? A) Theres no way to be sure you wont get HIV except to use condoms correctly. B) Only the correct use of a female condom protects against the transmission of HIV. C) There are new ways of protecting yourself from HIV that are being discovered every day. D) Other than abstinence, only the consistent and correct use of condoms is effective in preventing HIV.

D) Other than abstinence, only the consistent and correct use of condoms is effective in preventing HIV . Other than abstinence, consistent and correct use of condoms is the only effective method to decrease the risk of sexual transmission of HIV infection.

The nurse is completing a focused assessment addressing a patients immune function. What should the nurse prioritize in the physical assessment? A) Percussion of the patients abdomen B) Palpation of the patients liver C) Auscultation of the patients apical heart rate D) Palpation of the patients lymph nodes

D) Palpation of the patients lymph nodes During the assessment of immune function, the anterior and posterior cervical, supraclavicular, axillary, and inguinal lymph nodes are palpated for enlargement. If palpable nodes are detected, their location, size, consistency, and reports of tenderness on palpation are noted.

A 20-year-old man newly diagnosed with type 1 diabetes needs to learn how to self-administer insulin. When planning the appropriate educational interventions and considering variables that will affect his learning, the nurse should prioritize which of the following factors? A) Patients expected lifespan B) Patients gender C) Patients occupation D) Patients culture

D) Patients culture One of the major variables that influences a patients readiness to learn is the patients culture, because it affects how a person learns and what information is learned.

During the admission assessment of an HIV-positive patient whose CD4+ count has recently fallen, the nurse carefully assesses for signs and symptoms related to opportunistic infections. What is the most common life-threatening infection? A) Salmonella infection B) Mycobacterium tuberculosis C) Clostridium difficile D) Pneumocystis pneumonia

D) Pneumocystis pneumonia There are a number of opportunistic infections that can infect individuals with AIDS. The most common life-threatening infection in those living with AIDS is Pneumocystis pneumonia (PCP), caused by P. jiroveci (formerly carinii).

A nurses plan of care for a patient with rheumatoid arthritis includes several exercise-based interventions. Exercises for patients with rheumatoid disorders should have which of the following goals? A) Maximize range of motion while minimizing exertion B) Increase joint size and strength C) Limit energy output in order to preserve strength for healing D) Preserve and increase range of motion while limiting joint stress

D) Preserve and increase range of motion while limiting joint stress Exercise is vital to the management of rheumatic disorders. Goals should be preserving and promoting mobility and joint function while limiting stress on the joint and possible damage.

A patient with a sudden onset of hearing loss tells the nurse that he would like to begin using hearing aids. The nurse understands that the health professional dispensing hearing aids would have what responsibility? A) Test the patients hearing promptly. B) Perform an otoscopy. C) Measure the width of the patients ear canal. D) Refer the patient to his primary care physician.

D) Refer the patient to his primary care physician. Health care professionals who dispense hearing aids are required to refer prospective users to a physician if the patient has sudden or rapidly progressive hearing loss. This would be a health priority over other forms of assessment, due to the possible presence of a pathologic process.

A nurse is planning care for an older adult who lives with a number of chronic health problems. For which of the following nursing diagnoses would education of the patient be the nurses highest priority? A) Risk for impaired physical mobility related to joint pain B) Functional urinary incontinence related decreased mobility C) Activity intolerance related to contractures D) Risk for ineffective health maintenance related to nonadherence to therapeutic regimen

D) Risk for ineffective health maintenance related to non-adherence to therapeutic regimen For some nursing diagnoses, education is a primary nursing intervention. These diagnoses include risk for ineffective management of therapeutic regimen, risk for impaired home management, health-seeking behaviors, and decisional conflict.

A 40-year-old woman was diagnosed with Raynauds phenomenon several years earlier and has sought care because of a progressive worsening of her symptoms. The patient also states that many of her skin surfaces are stiff, like the skin is being stretched from all directions. The nurse should recognize the need for medical referral for the assessment of what health problem? A) Giant cell arteritis (GCA) B) Fibromyalgia (FM) C) Rheumatoid arthritis (RA) D) Scleroderma

D) Scleroderma Scleroderma starts insidiously with Raynauds phenomenon and swelling in the hands.

A patient with a diagnosis of retinal detachment has undergone a vitreoretinal procedure on an outpatient basis. What subject should the nurse prioritize during discharge education? A) Risk factors for postoperative cytomegalovirus (CMV) B) Compensating for vision loss for the next several weeks C) Non-pharmacologic pain management strategies D) Signs and symptoms of increased intraocular pressure

D) Signs and symptoms of increased intraocular pressure Patients must be educated about the signs and symptoms of complications, particularly of increasing IOP and postoperative infection.

The registered nurse taking shift report learns that an assigned patient is blind. How should the nurse best communicate with this patient? A) Provide instructions in simple, clear terms. B) Introduce herself in a firm, loud voice at the doorway of the room. C) Lightly touch the patients arm and then introduce herself. D) State her name and role immediately after entering the patients room.

D) State her name and role immediately after entering the patients room. There are several guidelines to consider when interacting with a person who is blind or has low vision. Identify yourself by stating your name and role, before touching or making physical contact with the patient. When talking to the person, speak directly at him or her using a normal tone of voice.

A patient is exploring treatment options after being diagnosed with age-related cataracts that affect her vision. What treatment is most likely to be used in this patients care? A) Antioxidant supplements, vitamin C and E, beta-carotene, and selenium B) Eyeglasses or magnifying lenses C) Corticosteroid eye drops D) Surgical intervention

D) Surgical intervention Surgery is the treatment option of choice when the patients functional and visual status is compromised. No nonsurgical (medications, eye drops, eyeglasses) treatment cures cataracts or prevents age-related cataracts.

A patients current antiretroviral regimen includes nucleoside reverse transcriptase inhibitors (NRTIs). What dietary counseling will the nurse provide based on the patients medication regimen? A) Avoid high-fat meals while taking this medication. B) Limit fluid intake to 2 liters a day. C) Limit sodium intake to 2 grams per day. D) Take this medication without regard to meals.

D) Take this medication without regard to meals. Many NRTIs exist, but all of them may be safely taken without regard to meals. Protein, fluid, and sodium restrictions play no role in relation to these drugs.

An occupational health nurse is in the planning stages of a new health promotion campaign in the workplace. When appraising the potential benefits of the program, the nurse should consider that success depends primarily on what quality in the participants? A) Desire to expand knowledge B) Self-awareness C) Adequate time- and task-management D) Taking responsibility for oneself

D) Taking responsibility for oneself Taking responsibility for oneself is the key to successful health promotion, superseding the importance of desire to learn information, self-awareness, or time-management.

A patients rheumatoid arthritis (RA) has failed to respond appreciably to first-line treatments and the primary care provider has added prednisone to the patients drug regimen. What principle will guide this aspect of the patients treatment? A) The patient will need daily blood testing for the duration of treatment. B) The patient must stop all other drugs 72 hours before starting prednisone. C) The drug should be used at the highest dose the patient can tolerate. D) The drug should be used for as short a time as possible.

D) The drug should be used for as short a time as possible. Corticosteroids are used for shortest duration and at lowest dose possible to minimize adverse effects.

A patient has come into the free clinic asking to be tested for HIV infection. The patient asks the nurse how the test works. The nurse responds that if the testing shows that antibodies to the AIDS virus are present in the blood, this indicates what? A) The patient is immune to HIV. B) The patients immune system is intact. C) The patient has AIDS-related complications. D) The patient has been infected with HIV.

D) The patient has been infected with HIV. Positive test results indicate that antibodies to the AIDS virus are present in the blood. The presence of antibodies does not imply an intact immune system or specific immunity to HIV.

A community health nurse is performing a visit to the home of a patient who has a history of rheumatoid arthritis (RA). On what aspect of the patients health should the nurse focus most closely during the visit? A) The patients understanding of rheumatoid arthritis B) The patients risk for cardiopulmonary complications C) The patients social support system D) The patients functional status

D) The patients functional status The patients functional status is a central focus of home assessment of the patient with RA.

A patient is diagnosed with giant cell arteritis (GCA) and is placed on corticosteroids. A concern for this patient is that he will stop taking the medication as soon as he starts to feel better. Why must the nurse emphasize the need for continued adherence to the prescribed medication? A) To avoid complications such as venous thromboembolism B) To avoid the progression to osteoporosis C) To avoid the progression of GCA to degenerative joint disease D) To avoid complications such as blindness

D) To avoid complications such as blindness The nurse must emphasize to the patient the need for continued adherence to the prescribed medication regimen to avoid complications of giant cell arteritis, such as blindness.

A school nurse is facilitating a health screening program among junior high school students. What purpose of health screening should the nurse prioritize when planning this program? A) To teach students about health risks that they can expect as they grow and develop B) To evaluate the treatment of students current health problems C) To identify the presence of infectious diseases D) To detect health problems at an early age so they can be treated promptly

D) To detect health problems at an early age so they can be treated promptly The goal of health screening in the adolescent population has been to detect health problems at an early age so that they can be treated at this time.

A nurse who provides care at the campus medical clinic of a large university focuses many of her efforts on health promotion. What purpose of health promotion should guide the nurses efforts? A) To teach people how to act within the limitations of their health B) To teach people how to grow in a holistic manner C) To change the environment in ways that enhance cultural expectations D) To influence peoples behaviors in ways that reduce risks

D) To influence peoples behaviors in ways that reduce risks The purpose of health promotion is to focus on the persons potential for wellness and to encourage appropriate alterations in personal habits, lifestyle, and environment in ways that reduce risks and enhance health and well-being.

A patient with chronic open-angle glaucoma is being taught to self-administer pilocarpine. After the patient administers the pilocarpine, the patient states that her vision is blurred. Which nursing action is MOST appropriate? A) Holding the next dose and notifying the physician B) Treating the patient for an allergic reaction C) Suggesting that the patient put on her glasses D) Explaining that this is an expected adverse effect

Pilocarpine, a miotic drug used to treat glaucoma, achieves its effect by constricting the pupil. Blurred vision lasting 1 to 2 hours after instilling the eye drops is an expected adverse effect.

A patient has a diagnosis of rheumatoid arthritis and the primary care provider has now prescribed cyclophosphamide (Cytoxan). The nurses subsequent assessments should address what potential adverse effect? A) Infection B) Acute confusion C) Sedation D) Malignant hyperthermia

When administering immunosuppressives such as Cytoxan, the nurse should be alert to manifestations of bone marrow suppression and infection.


Ensembles d'études connexes

RN Fundamentals Online Practice 2019 B with NGN

View Set

Computer Science 307 : Software Engineering : Chapter 5

View Set

VARCAROLIS Chapter 28: Child, Older Adult, and Intimate Partner Violence

View Set

ACCT 101A - Study for Exam II (Ch. 5)

View Set

The Gettysburg Address by Abraham Lincoln

View Set